You are on page 1of 48

PHARMACOLOGY, TOXICOLOGY, PHARMACOKINETICS

a. Only I and III are correct


1. Two dose-response curve response plotted on the same graph showed sigmoid curves having the same b. Only II, III and IV are correct
slope and height. T curve is located to the right of the A curve. Which of the following statements is/are c. Only II and IV are correct
correct regarding curves A an: d. Only III and V are correct
e. Only V is correct
3. The cumulative number of subjects who display a certain rersponse or effect to a drug is plotted on a graph
as shown below. The graph on the left represents beneficial effect while the graph on the right represents
toxic effect, what is the therapeutic index of the drug?

I. A is more effective than B


II. A is more potent than B
III. A and B have the same ceiling dose
IV. B is the graph produced when A is given with its competitive antagonist a.0.5 b. 1.5 c. 2.5 d.4.0 e. 5.0
V. B is the graph produced when A is given with its non-competitive antagonist 4. An administration of one drug leads to a shift in the log dose-response curve of another drug to the left a
range in the maximal efficacy, this is called
a. Only I and III are correct a. Agonism
b. Only II, III and IV are correct b. Partial agonism
c. Only II and IV are correct c. Non-competitive antagonism
d. Only III and V are correct d. Competitive antagonism
e. Only V is correct e. Potentiation
2. Two dose-response curve plotted on the same graph showed sigmoid curves A and B, where A is at the left 5. The action of this drug is dependent on a colligative property
of the graph with the height greater than B. Which of the following statements is/are correct regarding a. Mannitol
curves A and B? b. Cocaine
c. Reserpine
d. Furosemide
e. Losartan
6. Whiich of the following drug groups have mechanisms of aaction that involve binding to enzymes as target
protiens
a. Benzodiazepines
b. Barbiturates
c. Calcium channel blockers
d. Non-steroidal anti-inflammatory agents
e. Phenothiazine antipsychotics
7. Voltage-gated ion channels serve as site of actions for drugs such as
I. Diltiazem and Nifedipine
II. Lidocaine and Procaine
I. A and B has equal efficacy III. Captopril and Losartan
II. A and B have equal potency a. I only
III. B is a partial agonist of A b. I and II
IV. B is the graph produced when A is given with its competitive antagonist c. III only
V. B is the graph produced when A is given with its non-competitive antagonist
d. I and III b. Magnesium
e. I, II, and III c. Sodium
8. Drugs with mechanism of action that involves Tubulin-binding leading to loss of function of the structural d. Potassium
protien include e. Chloride
a. Cyclosporine 16. Which of the following stetements characterizes competitive antagonism?
b. Neostigmine a. There is a shift of the agonist log concentration-effect curve to the right and downwards
c. Colchicine b. There is a shift of the agonist log concentration-effect curve to the right without a change on the
d. Glibenclamide slope or amplitude
e. Metformin c. There is an exponential relationship between dose ratio and antagonist concentration
9. What type of protien is primarily targeted by Digitalis glycosides when they exert their effect on the heart? d. There is no competition for the binding sites
a. Receptors e. There is a shift of the agonist log-concentration effect curve to the left without a change in slope
b. Ion channels and amplitude
c. Enzymes 17. The minimum dose that produces the maximum achievable response is known as
d. Transporters a. Potency
e. Structural protiens b. ED50
10. Which of the following features cahracterize nicotinic, GABA-A, and Glutamate receptors? c. LD50
a. Thay control the movements of ions and out of the cell d. Ceiling dose
b. Their action is mediated bysecondary messengers e. Minimum effective dose
c. They all have excitatory effect on the cell membrane 18. The medium dose, or the dose of the drug that produces half of the maximum achievable response is called
d. They are located on the cytoplasm a. Potency d. Ceiling dose
11. Which of the following drugs bind to receptors located at the cell membrane? b. LD50 e. Binding constant
a. Thyroid hormone c. Therapeutic index
b. Prednisone 19. What is described by the equation given as follows:
c. Estrogen Response=f (EN total . Xa / (Xa + Ka)
d. Insulin a. Relationship between occupancy of receptor and the dose of the drug
e. Vitamin D b. Relationship between occupancy of receptor and response to the drug
12. Cyclic Adenosine Monophophate (cAMP) is generated as a secondary messenger by the action of the c. Relationship between the dose of the drug and the response to the drug
enzyme Adenylyl Cyclase. The substrate for the synthesis of cAMP is d. Relationship between the number of receptors and the dose of the drug
a. AMD e. Relationship between the response to the drug and the transducer function of the tissues
b. ADP 20. Competitive antagonist of the receptors of Dopamine and Benzodiazepine are expected to produced which
c. ATP effect?
d. GTP a. Reversal of the intrinsic activity of the receptors
e. Adenosine b. Maintain the consitutive activity of the receptors
13. Stimulation of Beta-adrenoceptor, a G-protien-linked receptor involves a generation of a secondary c. Stimulate the constitutive activity of the receptors
messenger known as d. Stimulate the constitutive activity of the recptorsin the absence of any ligand
a. IP3 e. Produce a negative efficacy
b. cGTP 21. The PT-INR of a patient with Warfarin has been maintained at 2.5 for the last 3 months. 1 week earlier, the
c. DAG patient was given Phenobarbital. What can be expected with the PT-INR of the patient?
d. cAMP a. Increased PT-INR
e. Calcium ions b. Decreased PT-INR
14. What is the mechanism of action of Sildenafil, a drug popularly known as viagra? c. Maintained PT-INR
a. Inhibits phosphodiesterase (PDE) leading to increase cAMP 22. Phenobarbital when given to a patient on chronic warfarin is classified as
b. Inhibits PDE leading to increase cGMP a. Pharmacodynamic antagonist
c. Stimulates adenylyl cyclase leading to increase cAMP b. Pharmacokinetic antagonist
d. Stimulates guanylyl cyclase leading to increase cGMP c. Physiologic antagonist
e. Inhibits phosphodiesterase leading to increase cGMP d. Synergistic agonist
15. Drugs that activate the phospholipase C-inositol phosphate system such as alpha-adrenergic agonists like e. Additive agonist
phenylephrine primarily cause an increase in the intracellular levels of which ion? 23. Digibind which effetively terminates the action of Digitalis glycosides is classified as what type of
a. Calcium antagonist?
a. Pharmacodynamic antagonist a. 0.2
b. Pharmacokinetic antagonist b. 2
c. Physiologic antagonist c. 4
d. Chemical antagonist d. 6
e. Non-competitive antagonist e. 8
24. Omeprazole can inhibiy gastric acid secretion induced by Histamine and Acetylcholine. It is thus classified 31. For drugs that are given constant rate, the time to reach steady rate state concentration dependent on
as what type of antagonist? a. Rate of administration
a. Pharmacodynamic antagonist b. Biologic half-life
b. Pharmacokinetic antagonist c. Volume distribution
c. Physiologic antagonist d. Peak plasma concentration achieved
d. Chemivall antagonist e. Amount of the drug in the body
e. Non-competitive antagonist 32. A given drug has a biologic half life of 4 hours. How much of the drug will remain in the body 12 hours
25. Which of the following mechanism may explain the development of tolerance to drugs? after a single dose?
a. Increase in the quantity of mediators a. 75%
b. Decreased metabolic degradation of the active drug b. 50%
c. Decreased excretion or extrusion of drugs from cells c. 25%
d. Downregulation of receptors d. 12.5%
e. Inactivation of counterregulatory physiologic response e. 6.25%
26. The prerequisite that drugs must be present in aqueous solution for them to be absorbed is true if the drugs 33. In compartmental analysis of serum drug concentration versus time plots, which of the following findings
are absorbed by which mechanism? confirm a one compartment model of drug behavior?
I. Passive transport a. An AUC above the explorated line that is less than 10% of the total AUC
II. Carrier-mediated transport b. An AUC above the explorated line that is less than 5% of the total AUC
III. Pinocytosis c. Slope of the last 3 terminal points differing by more than 10% for the slope of the first 3 terminal
a. I only points
b. I and II d. Slope of the 3 terminal points differing by less than 20% from the slope of the 3 terminal points
c. III only e. Cmax that is above the line explorated from the terminal points
d. I and III 34. What should be the rate of infusion of Theophylline if the desired plasma concentration is 10ug/mL in a 70
e. I, II, and III kg patient?Theophylline has a clearance of 0.6mL/minute per kg body weight.
27. According to the Fick’s Diffusion Equation, the rate of transport of a drug by passive diffusion is a. 6ug/min
a. Independent of the concentration gradient b. 4.2ug/min
b. Inversely proportional to the membrane surface area c. 17ug/min
c. Inversele proportional to the membrane thickness d. 420ug/min
d. Inversely proportional to the partition coefficient e. 1167ug/min
e. Independent of the diffusion constant of drug 35. A given drug with a half life of 8 hours is being administered at 100mg every 8 hours. What is the amount
28. Which of the following feature common to all carrier-mediated transport processes? remaining in the body after 32nd hour, just before administering the next dose?
a. Movement is along a concentration gradient a. 50mg
b. Involves non-specific binding to carriers b. 75mg
c. Can be characterized by allosteric inhibition c. 87.5mg
d. Displays a Michaelis-Menten Kinetics d. 93.75mg
e. Non-energy requiring e. 100mg
29. Which of the following substances may be absorbed significantly by pinocytosis 36. What is the con centration at steady state of Phenobarbital if the drug is given to a 60kg patient as a 65mg
a. Vitamin K tablet once daily with a bioavailability of 90%? The drug has volume distribution of 0.5liters/KBW and
b. Cyanocobalamin t1/2 of 100hrs
c. Cardiac glycosides a. 0.10mg/L
d. Quarternary ammonium compounds b. 1.50mg/L
e. Inorganic and organic electrolytes c. 11.70mg/L
30. 100ml of aqueous solutionof a given drug has a concentration of 25%.50ml of Octanol was added and after d. 22.20mg/L
equlibration, the concentration of the drug in the aqueous layer was 5%. What is the apparent lipid-water e. 48.20mg/L
partition coefficient of the drug?>
37. A 47 year old female diabetic is maintained on Tolbutamide for control of her hyperglycemia. Her fasting c. Deficiency of Glucuronosyl acyl transferase
blood sugar is maintained at 102mg%. Three days prior to admission, she self-medicated with d. Increased metabolism by nitro-reduction
Indomethacin due to a joint pain. She was admitted at the emergency room unconscious and difficult to e. Increased volume of distribution
arouse. What laboratory finding may be consistent with the history of the patient? 44. Cimetidine, Ketoconazole, and Allopurinol share a common pharmacokinetic property that predisposes
a. Capillary blood glucose of 20mg/dL patients taking other drugs concominantly with any of these agents to toxicity. What is this property?
b. Capillary blood glucose of 400mg/dL a. Inhibition of tubular secretion
c. ALT level that is 3x elevated b. Increased systemic absorption
d. ALT level that is 6x elevated c. Protien binding dsiplacement
e. Platelet level of 100,000/ul d. Enzyme induction
38. A drug molecule does not possess biologic activity by itself but it converted in the body to an active e. Enzyme inhibition
metabolite is called a/an 45. Biotransformation of orally administerd drugs occuring primarily in the liver that reduces the amount of
a. Orphan drug drug finally entering the systemic circulation is often referred to as __________.
b. Prodrug a. Enterohepatic recirculation
c. Prototype drug b. Gastric emptying
d. Parent drug c. Elimination
e. Metabolic drug d. First-pass effect
39. Astemizole and Terfenadine have been withdrawn from the market because of their intrinsic cardiotoxicity. e. Enzyme induction
Tha risk of cardiotoxicity is increased with concominant intake of Grapefruit juiceor Erythromycin. The 46. Charcoal-broiled foods, cruciferous vegetables and omeprazole induce the metabolism of drugs like
explanation for this interaction is tomoxifen, acetaminophen, theophylline and warfarin. This effect is due to induction of which CYP
a. Displacemnet from the protien binding of Astemizole or Terfenadine by Grapefruit juice of isoform?
Erythromycin a. CYP1A2
b. Decreased urinary excretion of Astemizole and Terfenadine b. CYP2A6
c. Inhibtion of the CYP3A4 enzyme by Grapejuice or Erythromycin c. CYP2C9
d. Increased absorption of Astemizole or Terfenadine d. CYP2D6
e. Induction of metabolism of Astemizole and terfenadineby Grapefruit juice or Erythromycin e. CYP4A4
40. Paients who are poor Debrisoquin metabolizers have decreased activity of which enzyme system 47. In a patient with hepatic encelopathy, which of the following agents may lead tosevere drug toxicity even
a. CYP 1A2 when given at usual doses?
b. CYP 2C9 a. Diazepam
c. CYP 2D6 b. Oral Neomycin
d. CYP 2C19 c. Lactulose
e. CYP 3A4 d. Propanolol
41. A patient has been assessed to have poor Debrisoquin metabolizing capability. Which of the following e. IV Gentamicin
drugs will have an increased likelihood of causing toxicity in this patient? 48. Which of the following features charcterizes the autonomic nervous system
a. Thioridazine a. Single neuron transmission
b. Amiodarone b. 0
c. Acetaminophen c. Effects are under volitional control
d. Warfarin d. Conveys outpus from the CNS to the skeletal muscles
e. Theophylline e. Terminal neurotransmitter is mainly Serotonin
42. What is considered as dominant phase I metaabolic reaction? 49. The preganglionic fibers of the sympathetic nervous system originate from which roots?
a. Oxidation a. Cranial nerves III, VII, IX, X
b. Reduction b. Sacral roots S1- S4
c. Hydrolysis c. Cervical roots C1-C7
d. Glucuronidation d. Thoracolumbar roots
e. Acetylation 50. What neurotransmitter is released by preganglionic fibers as a response to sympathetic stimulation?
43. Chloramphenicol when given to a nenonate or premature baby causes cardiovascular collapse manifestin as a. Acetylcholine
the so-called “Gray Baby Syndrome” This toxicity in such subsets of the patients is due to what age-related b. Norepinephrine
biologic feature? c. Epinephrine
a. Decreased protien binding d. Serotonin
b. Decreased urinary excretion of the metabolite e. Dopamine
51. What is the dominant ganglionic receptor found in most sympathetic fibers? d. Epinephrine
a. Alpha-1 e. Norepinephrine
b. Alpha-1 59. Reserpine, an alkaloid from Rauwolfia, can lower the blood pressure and depress the CNS. What enzyme
c. Beta-1 or part of the biosynthetic pathway is inhibited by Reserpine to bring about these effects?
d. Nicotinic a. DOPA decarboxylase
e. Muscarinic b. Storage of catecholamines in the presynaptic vesicles
52. What is the neurotransmitter mainly released by postganglionic parasympathetic fibers? c. Release of Norepinephrine from presynaptic terminals
a. Acetylcholine d. MAO enzymes
b. Norepinephrine e. Uptake-1 mechanism
c. Epinephrine 60. What is the most important mechansim of termination of effects of released Norepinephrine which is the
d. 5-Hydroxyryptamine target of action of tricyclic antidepressants and Cocaine
e. Dopamine a. Degradation of COMT
53. Which of the following non-adrenergic, non-cholinergic neurotransmitters is involved in the action of the b. Degradation of MAO
enteric nervous system? c. Sequestration into cells
a. ATP d. Reuptake into the presynaptic terminal
b. Dopamine e. Binding to presynaptic alpha-2 receptors
c. GABA 61. What is the main final metabolite of Norepinephrine and Epinephrine which when abnormally elevated is
d. Glutamate often used as a marker for the disease Pheochromocytoma?
e. Glycine a. Dihydroxymandelic acid
54. In synaptic neurotransmission, which of the following ions plays a significant role in neurotransmitter b. 3-Methoxy-4 hydromandelic acid
release c. 3-Methoxy-4 hydroxyphenylglycol
a. Sodium d. 3,4-Dihydroxyphenylglycol
b. Potassium e. Normetanephrine
c. Magnesium 62. Which of the following is an expected respose to alpha-1 adrenoceptor stimulation?
d. Calcium a. Uterine smooth muscle relaxation
e. Chloride b. Gastrointestinal sphincter constriction
55. The primary mechanism of neuro transmitter release is accomplished by what process? c. Ciliary muscle relaxation
a. Diffusion d. Gastrointestinal wall smoothe muscle contraction
b. Carrier-mediated e. Platelet aggregation
c. Active transport 63. A male patient wishes to purchase an over-the-counter tablet for colds. Which of the following data you
d. Facilitated transport gathered during an initial interview of the patient should prevent you from dispensing the medication?
e. Exocytosis I. The patient is currently taking Nifedipine GITTS
56. What is the amino acid precursor in the synthesis of catecholamines? II. The patient is currently on Tranylcypromine
a. Phenylalanine III. The patient is an elderly with history of urinary hesitancy, intermittency, and frequency
b. Glycine a. I only
c. Tyrosine b. I and II
d. Tryptamine c. III only
e. Glutamic acid d. I and III
57. In the biosynthesis of Norepinephrine, what step is considered rate-limiting? e. I, II, and III
a. Active uptake of Tyrosine 64. Stimulation of Alpha-1 receptors leads to the formation of important secondary messengers that promote
b. Conversion of tyrosine to dihydroxyphenylalanine calcium release and activation of protien kinase C. These secondary messengers responsible for the effects
c. Conversion of DOPA to dopamine of alpha-1 stimulation include which of the following?
d. Uptake of Dopamine in presynaptic storage vesicles a. Inositol triphosphate
e. Conversion of Norepinephrine to Epinephrine b. Cyclic adenosine monophophate
58. Which of the following substances exert a negative feedback control on Tyrosine Hydroxylase which c. Adenosine triphosphate
serves as the mechanis for moment-to-moment regulation of the rate catecholamine synthesis? d. Cyclic guanosine monophosphate
a. Dopamine e. Adenosine diphosphate
b. Acetylcholine 65. In which of the following clincal conditions are the agonists of the alpha-1 adrenoceptors found to be
c. DOPA useful?
I. Nasal congestion II. Alpha-1 activation leading to vasoconstriction
II. Hypotension III. Beta-2 activation leading to bronchodilation
III. Sympomatic bradycardia a. I only
a. I only b. I and II
b. I and II c. III only
c. III only d. I and III
d. I and III e. I, II, and III
e. I, II, and III 72. The activity of this drug is attributed to its metabolite which is a false neurotransmitter at the central
66. Which of the following effects can be attributed to alpha-2 receptor stimulation? nervous system
I. Vasodilation a. Guanethidine
II. Inhibition of catecholamine release b. Methyldopa
III. Vasoconstriction c. Propanolol
a. I only d. Labetalol
b. I and II e. Phenylpropanolamine
c. III only 73. Which of the following inotropic agents is most useful for paatient with acute heart failure complicated by
d. I and III acute reduction in creatinine clearance?
e. I, II, and III a. Epinephrine
67. A given adrenergic agent induces uterine relaxation and bronchial smooth muscle relaxation. These effects b. Norepinephrine
can be attributed to: c. Dopamine
a. Alpha-1 stimulation d. Dobutamine
b. Alpha-2 stimulation e. Digitalis
c. Beta-1 inhibition 74. In the adrenal medulla, norepinephrine is converted to epinephrine by the action of which enzyme?
d. Beta-1 stimulation a. DOPA decarboxylase
e. Beta-2 stimulation b. Tyrosine hydroxylase
68. What is the dominant adrenergic receptor in the heart? c. Cathecol-0- nethyl transferase
a. Dopamine-1 d. Monoamine oxidase
b. Alpha-1 e. Phenylethylamine N-methyl transferase
c. Alpha-2 75. As an inotropic agent, Norepinephrine (Levophed) has been shown in some small clinical trials to be most
d. Beta-1 useful in which of the following conditions?
e. Beta-2 a. Cardiogenic shock
69. Which of the following effects is/are consistent with the systemic use of the drug Clonidine? b. Septic shock
I. Initial transient increase in blood pressure c. Acute renal failure
II. “First-dose” phenomenon d. Acute myocarditis
III. Lowering of the blood pressure e. Dilated cardiomyopathy
a. I only 76. Which of the following statements is/are true regarding the drug methyldopa
b. I and II I. Is current clinical use is in the management of hypertension in pregnancy
c. III only II. It is associated with warm antibody type of immunohemolytic anemia
d. I and III III. Toxic doses are associated with hepatotoxicity
e. I. II, and III a. I only
70. In terms of mechanism of action, which of the following drugs most closely resembles that of Clonidine? b. I andII
a. Phenylphrine c. III only
b. Reserpine d. I and III
c. Guanethidine e. I, II, and III
d. Methyldopa 77. What are known clinical uses beta-2 agonists?
e. Amlodipine I. In the management of premature labor
71. Epinephrine is considered as the first line for the management of anaphylaxis and anaphylactic shock. II. As part of the interventions employed for hyperkalemia
Which of the following explain/s the mechanism of action of Epinephrine in anaphylaxis and anaphylactic III. Management of bronchial asthma
shock? a. I only
I. Inhibition of Histamine release by beta-2 receptor activation b. I andII
c. III only b. Nicotinic-N
d. I and III c. Presynaptic alpha-2
e. I, II, and III d. Postsynaptic alpha-2
78. Which of the following agents is classified as a selective direct-acting beta-2 agonist? e. Alpha-1
a. Propanolol 86. Which of the following is/are correct clinical indication for the drug Terbutaline
b. Mathoxamine I. Subcutaneously given as management of recalcitrant acute exacerbations of bronchial asthma
c. Isoxsuphrine II. To increase heart rate in symptomatic bradycardia
d. Prazosin III. Adjunct treatment of malignant hypertension
e. Ephedrine a. I only
79. During the conduct of ACLS, what is the recommended dosing ofor Epinephrine in adults? b. I andII
a. 1mg every 3-5 minutes c. III only
b. 3mg every 3-5 minutes d. I and III
c. Escalating dose, staring at 1mg then increasing by 1mg every 3-5 minutes e. I, II, and III
d. 10ml of a 1:1000 solution every 3-5 minutes 87. Which of the following drugs primarily stimulates beta-1 receptors?
80. Which of the following is an accepted indication for Epinephrine? a. Dobutammine
a. Bronchodilator fo intermittent bronchial asthma b. Ritodrine
b. Local vasoconstrictor c. Yohimbine
c. Anti-arrythmic for supraventricular tachycardia d. Phenoxybenzamine
d. Tocolytic agent for premature labor e. Oxymetazoline
e. Inotropic for cardiogenic shock 88. A new drug is currently being developed whose mechanism of action is stimulation of the beta-3 receptors.
81. A dominant alpha-1 agonist effect is expected when Epinephrine is given as an IV infusion at what doses? In which condition will this drug have most likely usefulness?
a. <10ng/kg/min a. Obesity
b. 10-50ng/kg/min b. Bronchial asthma
c. 10-80ng/kg/min c. Hypertension
d. At least 120ng/kg/min d. Shock
82. What is the clinical use of the Dopamine-1 agonist Fenoldopan? e. Acute renal failure
a. Chronic treatment of Congestive Heart Failure 89. A patient complains of muscle weakness after several doses of Salbutamol nebulization. This condition can
b. Acute treatment of Pheochromocytoma be attributed to _______________?
c. Acute treatment of Hypertension a. Hypokalemia induced by Salbutamol therapy
d. Chronis treatment of Supraventricular Tachycardia b. Hypocalcemia induced by Salbutamol therapy
e. Acute treatment of Congestive Heart failure c. Metabolic alkalosis due to bronchial asthma
83. Which of the following adrenergic agents is/are most useful for hypodynamic septic shock with d. Decreased firing of neuromuscular junction fibers as a direct effect of Salbutamol
hypotension e. Respiratory alkalosis due to bronchial asthma
I. Dopamine 90. Which of the following beta-blockers also has peripheral vasodilating effect similar to Prasozin?
II. Dobutamine a. Propanolol
III. Norepinephrine b. Betaxolol
a. I only c. Nadolol
b. I andII d. Bisoprolol
c. III only e. labetalol
d. I and III 91. Which of the following beta-blockers is considered cardio selective?
e. I, II, and III a. Labetalol
84. At what dose should Dopamine be given as IV infusion to produce a predominantly dopaminergic effect? b. propanolol
a. 0.5 ug/kg/min c. pindolol
b. 5-10ug/kg/min d. Esmolol
c. 10-20ug/kg/min e. Timolol
d. 20-50ug/kg/min 92. Rebound tachycardia and hypertension are expected complications of abrupt withdrawal of beta-blocker
e. >50ug/kg/min therapy. Which of the followingagents has the least likelihood of causing rebound effects when withdrawn?
85. Guanfacine and Guanabenz are primarily agonists at what receeptors? a. Labetalol
a. Muscarinic-1 b. Propanolol
c. Pindolol a. Continue her medication and suggest for a 12-lead ECG
d. Metropolol b. Stop Metoprolol and request for a 12-lead ECG
e. Timolol c. Stop Diltiazem and request for a 12-lead ECG
93. Which of the following beta-blockers has the shortest duration of action (half-life = 10 minutes) making is d. Stop indapamide and Diltiazem and request for a 12-lead ECG as well as serum potassium
useful for administration as continuous IV infusion with rapid achievemnet of a steady state concentration measurement
a. Metoprolol e. Stop Metoprolol and Diltiazem and request for a 12-lead ECG
b. Esmolol 99. Methylphenidate and Dextroamphetamine are centrally acting sympathomimetics which have found use in
c. Atenolol which of the following conditions?
d. Propanolol a. Manic-depression
e. Pindolol b. Alcohol withdrawal
94. Which of the following beta-blockers has/have been proven in clinical trials to be useful in the management c. Attention deficit hyperactivity disorder
os stable congestive heart failure d. Generalized anxiety disorder
I. Carvedilol e. Hypotensive epidoses
II. Bisoprolol 100. Which of the following effects is are consistent with direct stimulationof the muscarinic receptors?
III. Metoprolol I. Peripheral vasodilation
a. I only II. Relaxation of the ciliary muscles
b. I andII III. Negative dromotropic effect
c. III only a. a. I only
d. I and III b. I andII
e. I, II, and III c. III only
95. In which of the following conditions are beta-blockers found to be useful? d. I and III
I. Hypothyroidism e. I, II, and III
II. Raynaud’s Phenomenon 101. The effect of muscarinic receptor activation in tissues like the exocrine glands and the extravascular smooth
III. Initial therapy in Pheochromocytoma muscles have been linked with the generation of which second messenger molecule?
a. I only a. IP3
b. I andII b. cAMP
c. III only c. cGMP
d. I and III d. ATP
e. I, II, and III e. Adenosine
96. Among diabetic patients using insulin or oral hypoglycemic agents (OHAs) why are beta-blockers are used 102. What is thw physiologic event that follows stimulation of the N M type cholinergic receptors?
with special caution if not totally avoided? a. Opening of inward chloride channels
a. Beta-blockers can induce hyperglycemia b. Opening of outward potassium channels
b. Beta-blockers can mask hypoglemic symptoms c. Opening of inward sodium channels
c. Beta-blockers can attenuate effect of insulin and OHAs d. Opening of the outward calcium channels
d. Beta-blockers can induce insulin or OHA failure e. Opening of the outwad sodium channels
e. Beta-blockers can increase the risk of renal failure 103. Bethanecol is clinically used for which of the following conditions?
97. In which subset/s of patients are beta-blockers used with caution if not avoided? a. Urinary retention in neurogenic bladder
I. Patients with bronchospastic disease like COPD and bronchial asthma b. Intestinal obstruction
II. Patients with peripheral vascular disease especially vasospastic type c. Chronic glaucoma
III. Patients with recent myocardial infarction d. Supraventicular tachycardia
a. I only e. Congestive heart failure
b. I andII 104. What is/are the clinical use/s of Edorphonium (Tensilon)?
c. III only I. Reverse the effect of neuromuscular blockers after surgery
d. I and III II. Treatment of overdose of atropine
e. I, II, and III III. Management of organophosphate poisoning
98. A 44-year old patient is started a week ago on the following medications: Metoprolol 100mg BIB, a. I only
Diltiazem 30mb tablet BID and Indapamide 1.5 mg OD for poorly controlled hypertension and chronic b. I andII
stable angina pectoris. She is now admitted at the ICU for dizziness and two episodes of syncope. Her heart c. III only
rate was 45 beats/min. What course of action would you suggest? d. I and III
e. I, II, and III e. Inhibitors of bronchial gland secretions
105. Ambenonium, neostigmine, and pyridostigmine are most commonly used for which condition? 112. Oxynutinin, Dicyclomine, and Glycopyrrolate are clinically useful for which of the following conditions?
a. Atropine overdose a. Cholinomimetic agent overdose
b. Myasthenia gravis b. Gastrointestinal hypermotility and urinary incontinence
c. Bronchial asthma c. Angina due to coronary artery vasospasm
d. COPD d. Motion sickness and Parkinsonism
e. ADHD e. COPD and bronchial asthma
106. Tacrine, Donepezil, Rivastigmine and Galantamine are indicated for which of the following condition? 113. In most patients admitted to atropine overdose, most poison control experts recommend which of the
a. Supraventricular tachycardia following measures as primary management?
b. Curare-induced muscle paralysis I. Temperature control with cooling blankets
c. Accomodative esotropia II. Physostigmine as initial therapy
d. Alzheimer’s disease III. Diazepam for seizure control
e. Glaucoma a. I only
107. Which of the following agents maybe used for the management of glaucoma? b. I andII
I. Piocarpine-Physostigmine combination therapy c. III only
II. Ecothiophate d. I and III
III. Demecarium e. I, II, and III
a. I only 114. Poisoning resulting from ingestion of high dose quarternary antimuscarinic agents can be best managed
b. I andII with which of th following agents?
c. III only a. Physostigmine
d. I and III b. Neostigmine
e. I, II, and III c. Policarpine
108. Which of the following findings maybe consistent with an overdose of or accidental exposure to an d. Atropine
organophosphate e. Scopolamine
a. Constipation 115. Which of the following is/are important contraindication to the use of antimuscarinis agents or drugs with
b. Mydriasis atropine-like properties?
c. Peeripheral vasodilation I. History of prostatic hyperplasia
d. Emesis II. Glaucoma
e. Decreased sweating III. Hypertension
109. Which of the following effects can be attributed to atropine? a. I only
I. Ciliary muscle contraction b. I andII
II. Contraction of the papillary constrictor muscles c. III only
III. Blockade of vagal slowing of the heart d. I and III
a. I only e. I, II, and III
b. I andII 116. Trimethaphan is expected to produce which of the following effects?
c. III only a. Excessive sweating
d. I and III b. Pupillary constriction
e. I, II, and III c. Diarrhea
110. In comparison to other aantimuscarinic agents, Scopolamineis expected to exert significant CNS effects d. Urinary incontinence
due to: e. Hypotension
a. Its being a tertiary amine 117. What is the mechanism of skeletal muscle paralysis produces by Succinylcholine?
b. Its being a quarternary amine a. Competitively inhibits binding of acetylcholine to the NM receptors
c. The presence of a benzene ring structure b. Increased inward chloride conductance leading to hyperpolarization of the neuromuscular endplate
d. The tropic acid structureits isomeric structure membrane
111. Ipratropium, Oxytropium, and Tiotropium and antimuscarinic agents that are most useful as c. Sustained depolarization of the motor end plate making it unesponsive to other impulses upon
a. Mydriatics repolarization cannot easily depolarized
b. Cycloplegic agents d. Repeated short depolarizations of the motor end plate leading to prolonged state of sodium
c. Relaxants of bronchial smooth muscles channel inactivation
d. Inhibitors of involuntary muscle contractions
e. Irreversible and non-competitive inhibition of the NM receptors in both the ganglia and motor end b. I andII
plate with attenuation of depolarizing impulses c. III only
118. Which of the following findings is consistent with the skeletal muscle relaxant effect of Pancuronium and d. I and III
Rocuronium? e. I, II, and III
a. Effect is enhanced with administration of succinylcholine 125. Which of the following drugs inhibit enzyme carbonic anhydrase?
b. Effect is reduced with co-administration of tubocurarine a. Dichlorphenamide
c. Initially cause depolarization of the motor end plate leading to fassiculations b. Tolazamide
d. Effect is antagonized by Neostigmine c. Astemizole
e. Sustained response to tetanic stimulation d. Selegiline
119. Which of the following inhilational anesthetics produces greatest augmentation of the effect of e. Donepezil
neuromuscular blockers? 126. What is the primary mechanism of action of the drug Indapamide?
a. Isoflurane a. Inhinition of the 2Cl-Na-K cotransporters in the convoluted tubule
b. Sevoflurane b. Inhibition of the Na-Cl contransporters at the convoluted tubule
c. Halothane c. Inhibition of the Na-Cl cotransporters at the collecting duct
d. Desflurane d. Competitive inhibition of aldosterone binding to its receptors
e. Nitrous oxide e. Competitive inhibition os vasopressin binding to V2 receptors
120. Anaphylactoid reaction may be seen as a side effect with which of the following neuromuscular blockers? 127. Which side effect of chlorthalidone is prominent especially among the elderly when Chlorthalidone is
a. Pancuronium given at a dose of at leats 50mg/day and significantly reduced when doses are maintained below 50mg/day?
b. Vecuronium a. Hyperuricemia
c. Gallamine b. Hyperglycemia
d. Tubocurarine c. Increased in serum LDL
e. Rocuronium d. Allergic reactions
121. Which of the following statements correctly describes the side effects of succinylcholine? e. Hyponatremia
I. Negative chonotropic and inotropic effects which can be attenuated by atropine 128. What is the rationale behind the combination of amiloride and Hydrochlorothiazide (HCTZ)
II. Increase risk of emesis among diabetics and obese patients a. Minimize hyperglycemia due to HCTZ
III. Hyperkalemia when given to burn or trauma patients or patients with close head injury b. Reduce risk of renal stone formation with HCTZ
a. I only c. Reduce hyperkalemia with amiloride
b. I andII d. Minimize bicarbonaturia with HCTZ
c. III only e. Minimize hypokalemia due to HCTZ
d. I and III 129. The action of the drug Eplerenone most closely resembles which of the following agents?
e. I, II, and III a. Spironolactone
122. Which of the following mechanism may explain the ability of beta-blockers to reduce blood pressure in b. Amiloride
hypertension? c. Triamterence
a. Increased compliance of the capacitance vessels d. Metolazone
b. Relaxation of the primary resistance vessels e. Quinethazone
c. Inhibition of renin release 130. Which of the following agents exhibits pharmacodynamic interaction with the thiazide diuretics leading to
d. Direct inhibition of Angiotensin II receptors a reduced diuretic effect?
e. Decrease in total body stores of sodium a. Furosemide
123. Which of the following equations describe the factors that contribute to blood pressure? b. Amiloride
a. BP=SV/SVR c. Enalapril
b. BP=COxSVR d. Hydralzine
c. BP=SVxHR e. Digoxin
d. BP=CO/SVR 131. Which of the following mechanism explain the effects of the drug Torsemide and Bumetanide?
e. BP=SVxSVR a. Inhibition of carbonic anhydrase at the proximal convoluted tubule
124. Which of the following mechanism explains the blood pressure-lowering effect of thiazide diuretics? b. Inhibition of the Na-K-2Cl co-transporter at the thick ascending limb of the Loop of Henle
I. Reduction in blood volume and cardiac output c. Inhibition of the Na-Cl co-transporter at the thin ascending limb of the Loop of Henle
II. Increased vascular compliance d. Inhibition of the Na-Cl co-transporter at the distal convoluted tubule
III. Decreased proximal tubular reabsorption of water and sodium e. Inhibition of the Na-Cl co-transporter at the distal tubule and collecting duct
a. I only 132. Which of the following is/are accepted indication/s for the drug Spironolactone?
I. Adjunct in the management of CHF a. The molecule releases norepinephrine in the presynaptic vesicles and is released by nerve
II. Conn’s Syndrome stimulation to interact with postsynaptic adrenergic receptors
III. Nephrogenic Diabetis Insidipus b. The molecule is converted first to alpha-methyldopamine and alpha-methylnorepinephrine which
a. I only replace norepinephrine in the presynaptic vesicles and are released by nerve stimulation to interact
b. III only with postsynaptic adrenergic receptors
c. I and II c. The molecule is converted first to alpha-methyldopamine and alpha-norepinephrine which
d. II and III stimulate central alpha receptors
e. I, II, and III d. The molecule is converted first to alpha-methyldopamine and alpha-norepinephrine which inhibit
133. Which of the following is/are accepted indication/s for the drug Furosemide? central alpha receptors
I. Infusion-diuresis in hypercalcemia e. The molecule directly inhibits peripheral alpha and beta receptors
II. Oliguric acute renal failure 139. What is the most frequent side effect of methyldopa which can be seen particularly at the onset of
III. Acute pulmonary edema treatment?
a. I only a. Sedation
b. III only b. Depression
c. I and II c. Extrapyramidal signs
d. II and III d. (+) coomb’s test
e. I, II, and III e. Galactorrhea
134. Which of the folling agents is primarily indicated for the management of increased intracranial pressure 140. A 45 years old female in clonidine 1.0mg/day for thee last ffour weeks was brought to the hospital for 1
a. Mannitol day history of nervousness, tachycardia and abrupt increase in her blood pressure to 180/130mmHg. Her
b. Acetazolamide last intake of clonidine was last 2 days prior to admission. Which of the following is/are appropriate
c. Furosemide interventions that you can suggest for the patient?
d. Chlorthalidone I. Start patient on Propanolol 50mg BID
e. Bumetanide II. Re-institute Clonidine 1.0mg/day
135. Which of the following agents is most useful for the management of nephrolithiasisndue to idiopathic III. Start Labetalol 200mg/day
hypercalciuria? a. I only
a. Ethacrynic acid b. III only
b. HCTZ c. I and II
c. Dorzolamide d. II and III
d. Vasopressin 141. Which of the following side effects is/ are common to the three drugs: Hydralazine, Diazoxide, and
e. Triamterene Minoxidil?
136. Which of the following statements correctly describes the characteristics of Indapamide as a drug for a. Hypertrichosis
hypertension? b. Glucose intolerance
I. Its diuretic effect is self-limiting, and may not account for its hypertensive effect after about 2-4 c. Reflex tachycardia
weeks of therapy d. Lupus-like side effect
II. Long-term use leads to improved blood vessel compliance e. Dyslipidemia
III. Has modest vasodilating property 142. In which subset of patients should Hydralazine be avoided or used with caution?
a. I only a. Patients with arrythmias
b. II only b. Patient with hypertension
c. I and III c. Patients with DM
d. II and III d. Patients with Ischemic Heart disease
e. I, II, and III e. Patients with CHF
137. Which of the following agents maybe associated with ototoxicity which may have increased risk of 143. Which of the following drugs if combined with Minoxidil may minimize latter drug’s side effect?
occuring when patient also recieve aminoglycoside antibiotic? a. Hydralazine+HCTZ
a. Furosemide b. Guanethidine+Reserpine
b. Chlorthalidone c. Metoprolol+Chlorthalidone
c. Dorzolamide d. Amlodipine
d. Amiloride e. Amlodipine+Prazosin
e. Spironolactone 144. In the management of Pparoxysmal hypertensionin pheochromocytoma, which of the following drugs is the
138. Which of the following mechanisms explains best the action of drug methyldopa? most appropriate initial therapy?
a. Phentolamine II. Diazoxide
b. Metoprolol III. Sodium nitropusside
c. Guanfacine a. I only
d. Reserpine b. II only
e. Clonidine c. I and II
145. Orthostatic hypotension and syncope are manifestations of a phenomenon associated with the first dose of d. II and III
which agent? e. I, II, and III
a. Labetalol 152. The decrease in arterial tone due to vasorelaxant action of sodium nitropusside is associated with
b. Methyldopa a. Increase in smooth muscle cGMP levels
c. Fenoldopam b. Increase in smooth muscle cAMP levels
d. Irbesartan c. A decrease in calcium entry through L-type channels
e. Doxazosin d. A local anethetic effect on smooth muscle cells
146. The mechanism of action of the drug clonidine most closely resembles that of ______________? e. Stimulation of inward K-channel
a. Methyldopa 153. Whgich of the following effects can be attributed to Angiotensin II?
b. Propanolol I. Stimulation of release of nitric oxide
c. Carvedilol II. Stimulation of synthesis and release of Aldosterone
d. Guanethidine III. Stimulation of release of Norepinephrine
e. Reserpine a. I only
147. Constipation, Urinary retention, tachycardia, mydriasis and anhidrosis are expected effects based on the b. II only
mechanism of action of which of the following drugs? c. I and II
a. Clonidine d. II and III
b. Trimethaphan e. I, II, and III
c. Reserpine 154. What is the primary advantage of Angiotensin II receptor blockers over the ACE-Inhibitors?
d. Sodium nitropusside a. Lesser incidence of coughs
e. Nicardipine b. Absence of risk interstitial nephritis
148. Which of the following drugs may be used safely for the control of hypertension in pregnant patients with c. Safe for pregnant patients with hypertension
pre-eclampsia? d. Faster onset of action
I. Magnesium Sulfate e. Lesser incidence of hyperkalemia
II. Methyldopa 155. Which of the following ACE-Inhibitors require hydrolysis to the active metabolite/s in the body to produce
III. Hydralazine effects?
a. I only I. Enalapril
b. II only II. Ramipril
c. I and II III. Perindopril
d. II and III a. I only
e. I, II, aand III b. II only
149. Which of the following mechanisms explains the action of the drug Sodium Nitropusside? c. I and II
a. Activation of guanylyl cyclase with increase of cGMP d. II and III
b. Release of N20 with subsequent increase in cAMP e. I, II and III
c. Inhibition of the secretion of renin 156. In which of the following conditions are ACE-inhibitors found to be clinically useful as first-line
d. Stimulation of A1 receptors treatment?
e. Increase conductance of outward potassium channels I. Angina pectoris
150. Which of the following agents is pure anteriolar vasodilaator? II. CHF
a. Prazosin III. Diabetic Nephropathy
b. Sodium Nitropusside a. I only
c. Diazoxide b. II only
d. Phentolamine c. I and II
e. Pindolol d. II and III
151. Which of the following agents can be used for hypertensive emergencies e. I, II, and III
I. Enalaprilat
157. The efficacy of ACE-inhibitors in the management of hypertension may be reduced in the presence of c. Metoprolol
certain drugs such as ___________? d. HCTZ
a. Ibuprofen e. Enalapril
b. Felodipine 164. Which of the following anti-hypertensive agents may be more appropriate to use for diabetic patients with
c. Indapamide evidence of renal disease?
d. Eprenolone a. Prazosin
e. Hydralazine b. Amlodipine
158. In which subset of patients which are ACE-inhibitors avoided as treatment option? c. Metoprolol
I. Patients with bilateral renal artery stenosis d. HCTZ
II. Patients with hyperkalemia e. Enalapril
III. Pregnant patients in their 2nd and 3rd trimesters of pregnancy 165. Chinese patients are most likely to be more sensitiveto effects of which of the following drugs for
a. I only hypertension that lower doses may have to be used?
b. II only a. Prazosin
c. I and II b. Amlodipine
d. II and III c. Metoprolol
e. I, II, and III d. HCTZ
159. Which of the following calcium channel blockers has greater activity on calcium channels in myocardial e. Enalapril
tissues than calcium channels in the vascular smooth muscles? 166. In the management of hypertensive emergencies, which of the following goals or interventions is/are
a. Diltiazem correct?
b. Verapamil I. Rapid normalization of the blood pressure within 24 hours
c. Nifedipine II. Lowering of the mean arterial pressure by about 25%
d. Amlodipine III. Initiation of oral hypertensives early as soon adequate blood pressure lowering has been achieved
e. Felodipine a. I only
160. What is the mechanism of action of the diydropyridine calcium channel blockers? b. II only
a. Inhibit the L-type calcium channels in the arteriolar smooth muscles c. I and II
b. Inhibit the L-type calcium channels in the anteriolar and venous smooth muscles d. II and III
c. Equally inhibit the L-type calcium channels in the vascular and cardiac tissues e. I, II, and III
d. Inhibit the S-type calcium channels in the anteriolar smooth muscles 167. Which of the following are important considerations regarding the proper handling of Nitroglycerin
e. Inhibit the S-type calcium channels in the anteriolar and venous smooth muscles tablets
161. In the long-term management of hypertensiion, which of the following agents is/are considered as I. Formulations used in the medicine are explosive
appropriate maintenance therapy? II. Sublingual tablets lose their potency when stored in plastic containers
I. Regular Nifedipine capsules III. These are sensitive to light
II. Enalapril tablets a. I only
III. Indapamide tablets b. II only
a. I only c. I and II
b. II only d. II and III
c. I and II e. I, II, and III
d. II and III 168. Which of the availabe nitrovasodilators has a bioavailabilty of about 100%?
e. I, II, and III a. Isosorbide mononitrate
162. What is the most common side effect of Enalapril? b. Isosorbide dinitrate
a. Angioedema c. Amyl nitrite
b. Interstitial nephritis d. Glyceryl trinitrate
c. Coughs e. None aqs all undergo significant first pass effect when given orally
d. Hypotension 169. Which of the following statements most closelydescribes the primary mechanism of action of the
e. Drug fever nitrovasodilator?
163. Which of the following anti-hypertensive agents is the most appropriate for elderly male patients with a. Denitrification by glutathione S-transferase leads to release of nitrite ion which is then converted
benign prostatic hyperplasia? to nitric oxide, a molecule that activates adenylyl cyclase leading to increase cAMP
a. Prazosin b. Denitrification by glutathione S-transferase leads to release of nitrite ion which is then converted
b. Amlodipine to nitric oxide, a molecule that activates guanylyl cyclase leading to increase cGMP
c. Nitrate ion released upon denitrification directly stimulas outward K-channels leading to a. I only
hyperpolarization of the vascular smooth muscles b. II only
d. Nitrate ion released upon denitrification directly stimulas outward K-channels leading to c. I and II
hyperpolarization of the vascular smooth muscles d. II and III
e. Nitrate ion released upon denitrification stimulates the production of prostacylin leading to e. I, II, and III
vasodilation 176. What is the primary limitation with the use of Dihydropyridine Calcium Channel Blockers in the
170. Which of the following tissues exhibit the greatest sensitivity to nitrovasodilators at the lowest effective management of angina pectoris?
doses? a. Tendency to reduce cardiac preload
a. Arteries b. Tendency to cause reflex tachycardia
b. Arterioles c. Tendency to induce reflex vasospam when stopped
c. Capillaries d. Tendency to cause peripheral edema
d. Veind e. Tendency to reduce cardiac end-diastolic pressure
e. Myocardial tissues 177. What is the most important role of beta-blockers in the management of angina pectoris
171. Which of the following statements correctly explains the primary mechanism of relief of angina pectoris a. Causes coronary artery vasodilatation thus useful in the management of acute anginal attacks
with the use of nitrovasodilators when given at usual doses? b. Dilates the epicardial coronary vessels thus useful in patients with ongoing prinzmetal angina
a. Peripheral dilation of the veins leading to reduction in cardiac preload and myocardial oxygen c. Reduces myocardial oxygen demand by reducing the total peripheral resistance making it useful
demand for chronic stable angina pectoris
b. Vasodilation of epicardial coronary arteries leading to improved oxygen delivery to myocardial d. Reduces myocardial oxygen demand by reducing myocardial perfusion time making it useful for
tissues chronic stable angina pectoris
c. Increase in diastolic perfusion time leading to improved perfusion to ischemic myocardium e. Reduces myocardial oxygen demand by decreasing myocardial perfusion time making it useful for
d. Decrease total peripheral resistance with arteriolar vasodilation all types of angina pectoris
e. Decrease in myocardial contractility leading to reduction in myocardial oxygen demand 178. Which of the following drugs or drug combination is/are useful in the management of chronic stable
172. Which of the following agents classified as nitrovasodilators is useful in the management of cyanide angina pectoris?
poisoning? I. Regular release Nifedipine capsule
a. Sodium nitrite II. Metoprolol tablet
b. Amyl nitrite III. Metoprolol tablet + Diltiazem tablet
c. Sodium thiosulfate a. I only
d. Nitroglycerin b. II only
e. Isosorbide mononitrate c. I and III
173. Which of the following nitrovasodilators may induce conversion of hemoglobin to methemoglobin d. II and III
(methemoglobinemia) which can lead to cyanosis? e. I, II, and III
a. Nitroglycerin 179. What is the principal mechanism of action of cardiac Glycosides?
b. Isosorbide mononitrate a. Stimulate the release of calcium for the sarcoplasmic reticulum leading to release in an increase
c. Isosorbide dinitrate intracellular calcium
d. Tetrathyl trinitrate b. Inhibition of the cell membrane outward calcium transporter preventing calcium extrusion
e. Amyl nitrite c. Stimulation of inward calcium transporter leading to an increase in intacellular calcium
174. This side effect of nitrovasodilators is explained in part by the depletion of sulfhydryl moieties in vascular d. Inhibition of the Na-K-ATPase leading to increase in intracellular sodium and preventing calcium
smooth muscles with continuous use of the drugs, and may be partly reversed bysulfhydryl-regenerating extrusion
compunds. e. Inhibition oh the Na-Ca exchanger in the sarcoplasmic reticulum leading to increase in
a. Tolerance intracellular calcium
b. Increased intracranial pressure 180. Which of the following statements on pharmacokinetic parameters must be considered for patients taking
c. Reflex tachycardia Digoxin?
d. Methemoglobenia a. In about 10% of patients, use of antibiotics can lead to increased oral bioavailability
e. carcinogenicity b. Renal clearance of the drug is inversely proprtional to the creatinine clearance
175. Which of the following agents is/are considered useful as single agent in the acute treatment of vasospastic c. Half-life is long, at about 168 hours due in part to extensive enterohepatic recirculation
angina pectoris? d. Hepatic metabolism accounts for more than 80% of the elimination of the drug
I. Nitrovasodilators e. It is primarily distributed within the intravascular space due to its high protein binding
II. Beta-blockers 181. Which of the following is/are mechanical effects of digoxin on myocardial tissues
III. Calcium channel blockers I. Increase in contractility of myocardial tissues
II. Increase in conduction velocity in the AV node d. Suggest discontinuing digoxin and furosemide and checking for serum K levels for necessary
III. A positive inotropic effect correction
a. I only e. Suggest discontinuing all medications and starting patient instead on an ACE inhibitor, vasodilator
b. II only and a thiazide
c. I and III 187. Which of the following intervention may be appropriate in the management of severe cases of digitalis
d. II and III intoxicationsuch as in children and in suicidal overdose?
e. I, II, and III I. Administration of digoxin immune fab
II. Aadministration of KCl
182. Which of the following is/are expected electrical effect/s of digoxin on myocardial tissues? III. Administration of MgSO4
I. Decrease in the refractory period of atrial muscles a. I only
II. Increase in the refractory period of the AV node b. II only
III. A positive inotropic effect c. I and III
a. I only d. II and III
b. II only e. I, II, and III
c. I and III 188. What is considdered as the toxic plasma concentration of digoxin?
d. II and III a. >0.5mg/mL
e. I, II, and III b. >150mgng/mL
183. Which of the following condition may augment the effect of digitalis glycosides in myoccardial tissues? c. >10ng/mL
I. Concurrent use of captopril d. >5ng/mL
II. Reduced oxygenation of myocardial tissues e. >2ng/mL
III. Concurrent administration of calcium gluconate 189. What is the primary use of drugs like Inamirone and Milrinone?
a. I only I. Alternative to digoxin in patients with chronic CHF
b. II only II. Management for acute heart failure
c. I and III III. Management of exacerbation of chronic CHF
d. II and III a. I only
e. I, II, and III b. II only
184. Which of the following is/are the primary indication/s of Digitalis Glycosides? c. I and III
I. First line drug for the initial management of CHF d. II and III
II. Useful in controlling the rate of ventricular response in atrial fibrillation e. I, II, and III
III. Useful in pregnant patients with CHF 190. What is the mechanism of action of Inamrinone and Milrinone in the management of heart failure?
a. I only a. Increase the levels of cAMP by activating the enzyme adenylyl cyclase
b. II only b. Increase the levels of cAMP by inhibiting the enzyme phosphodiesterase III
c. I and III c. Increase the levels of cGMP by inhibiting the enzyme guanylyl cyclase
d. II and III d. Inhibit the outward calcium flux leading to increased intracellular calcium
e. I, II, and III e. Primarily cause peripheral vasodilation leading to reduced cardiac preload
185. What is the major mechanism of interaction between Digoxin and Quinidine? 191. Inotropic activity oh the heart in CHF can be increased by which of the following agents?
a. Displacement of digoxin from tissue binding site I. Bipyridines
b. Decreased hepatic metabolism of digoxin II. ACE inhibitors
c. Decresed renal clearance of digoxin III. Beta-1 agonists
d. Increase binding of digoxin with ion transporters in the myocardium a. I only
e. Increase intestinal absorption of digoxin b. II only
186. A 50 years old male patient with CHF was started on the following medications: digoxin 0.25 mg OD, c. I and III
Hydralazine 10mg BID, furosemide 20mg OD, and ISMN 60mg OD. 1 week after initiating therapy, the d. II and III
patient was admitted for blurring of vision and palpitation. He feels nauseated and has problems e. I, II, and III
discriminating colors. Which of the following sourses of action is appropriate? 192. Which of the following agents for the heart failure may be associated with arrythmia, thrombocytopenia,
a. Suggest discontinuing Hydralazine and starting patient on amlodipine aand hepatotoxicity which limit their use in patients with chronic CHF?
b. Suggest giving of metoclopramide for the nausea and examining the probable eye problem A. Bipyridines
c. Suggest starting patient on beta-blocker therapy B. Digitalis glycosides
C. Beta-1 agonists
D. ACE inhibitors III. Propanolol
E. Angiotensin II antagonists a. I only
193. Which of the following statements correctly describe/s the appropriate use of drugs for CHF b. II only
I. Beta-blockers when used for patients with stable CHF can be given at doses similar to the dose c. I and III
used for hypertension d. II and III
II. ACE inhibitor dose must be carefully titrated to the maximum allowable or tolerable dose e. I, II, and III
III. Spironolactone has been found to be most useful as initial drug therapy for CHF even prior to the 199. Which of the following anti-arrythmic agents has been associated with lupus-like side effects?
use of ACE inhibitors or casodilators a. Quinidine
a. I only b. Lidocaine
b. II only c. Propafenone
c. I and III d. Amiodarone
d. II and III e. Procainamide
e. I, II, and III 200. What is the drug of choice for the management of sustained ventricular arrythmia associated with acute
194. Which of the following drugs or drug combinations can reduce both the cardiac preload and afterload in myocardial infarction?
CHF? a. Procainamide
I. Hydralazine + Nitrovasodilators b. Lidocaine
II. Beta-blockers + Loop diuretics c. Amiodarone
III. Loop diuretics + Digitalis d. Verapamil
a. I only e. Adenosine
b. II only 201. What is the preferred drug for the management of ventricular arrythmias associated with digitalis
c. I and III intoxication?
d. II and III a. Procainamide
e. I, II, and III b. Lidocaine
195. Which of the following interventions may be beneficial in the management of CHF? c. Amiodarone
I. Replacement of chronic digoxi dose with ACE inhibitors d. Verapamil
II. Add-on therapt with Aldosterone antagonist on patients already on digoxin or ACE-inhibitors e. Adenosine
III. Use of calcium channel blockers in patient who cannot recieve beta-blockers 202. What is the preferred drug for the management of acute episodes of supraventricular tachycardia?
a. I only a. Quinidine
b. II only b. Verapamil
c. I and III c. Amiodarone
d. II and III d. Bretylium
e. I, II, and III e. Adenosine
196. Which of the following drugs is a sodium channel blockers that can prolong the duration of the action 203. What is the preferred drug for the management of chronic paroxysmal supreventricular tachycardia?
potential and dissociates from the channel with intermediate kinetics? a. Quinidine
a. Lidocaine b. Verapamil
b. Flecainide c. Amiodarone
c. Quinidine d. Bretylium
d. Bretylium e. Adenosine
e. Diltiazem 204. What is the primary mechnism of action of the drug Amiodarone?
197. Which of the following drugs is a sodium channel blockers which has no significant effect on the duration a. Prolong the action potential with block of the sodium channels
of the action potential and dissociate from the channel with slow kinetics? b. Shorten the action potential with block of the sodium channels
a. Lidocaine c. Prolong the action potential with block of the potassium channels
b. Flecainide d. Shorten the action potential with block of the potassium channels
c. Quinidine e. Inhibit intrinsic sympathetic stimulation by preventing activation of the enzyme adenylyl cyclase
d. Bretylium 205. What side effect/s may be expected with the use of Amiodarone?
e. Sotalol I. Hypothyroidism or Htperthyroidism
198. Which of the following beta-blockers have been found to be most useful as anti-arrythmic agent/s? II. Fatal pulmonary fibrosis
I. Esmolol III. Symptomatic bradycardia and heart block particularly in patients with atrioventricular nodal
II. Metoprolol disease
a. I only b. Phenothiazine
b. II only c. Piperazine
c. I and III d. Ethanolamine
d. II and III e. Ethylaminediamine
e. I, II, and III 212. Which of the following agents is used primarily for its sedating property as the active ingredient of an
206. Measurement of the serum ALT prior to and during therapy of arrythmia is indicated when which of the OTC sleeping aid called Unisom?
following drugs for arrythmia is/are used? a. Carbinoxamine
a. Propafenone b. Doxylamine
b. Moricizine c. Hydoxyzine
c. Amiodarone d. Meclizine
d. Flecainide e. Tripelenamine
e. Diltiazem 213. What class of H1-antihistamines include drugs that sre used as components of OTC “cold” medications
207. Which of the following calcium channel blockers is most useful in arrythmia therapy? a. Alkylamines
a. Verapamil b. Phenothiazine
b. Diltiazem c. Piperazine
c. Nifedipine d. Ethanolamine
d. Lidocaine e. Ethylaminediamine
e. Propafenone 214. What is the active metabolite of hydroxyzine which is also available as an H1-antihistamine without
208. Which of the following effects can be attributed to histamine binding to its H1 receptor? significant sedating side-effect?
I. Extravascular smooth muscle contraction a. Fexofenadine
II. Vascular smooth muscle contraction b. Astemizole
III. Gastric acid secretion c. Cetirizine
a. I only d. Loratidine
b. II only e. Terfenadine
c. I and III 215. Which of the following H1 anti-histamine chemical groups possess significant atropine-like effect?
d. II and III a. Alkylamines
e. I, II, and III b. Phenothiazine
209. The triple response of Lewis following intradermal injection of Histamine includes which of the c. Piperazine
following? d. Ethanolamine
I. Sensory nerve ending stimulation producing flare e. Piperidines
II. Erythema caused by contraction of vascular smooth muscles 216. Which of the following H1-antihistamines has significant blocking effects on 5-HT receptors?
III. Endothelian cell contraction leading to exudation of fluids a. Promethazine
a. I only b. Cyproheptadine
b. II only c. Pyrilamine
c. I and III d. Carbinoxamine
d. II and III e. Cetirizine
e. I, II, and III 217. Which of the following H1-antihistamines has significant anti-cholinergic effect to be useful for the
210. Which of the following effects may be expected as a direct consequence of blockade of the H1 receptots management of dytonia and pseudoparkinsonism associated with the use of neuroleptic agents?
I. Sedation a. Diphenhydramine
II. Resolution of initial allergic response b. Brompheniramine
III. Anti-nausea and Anti-emetic effect (anti-motion sickness) c. Promethazine
a. I only d. Cyproheptadine
b. II only e. Cyclizine
c. I and III 218. Which of the following H1-antihistamines can block sodium channels in excitable membranes bringing
d. II and III about local anesthetic effect?
e. I, II, and III a. Diphenhydramine
211. Dimenhydrinate and Diphenhydramine are H1 antihistamines that belong to what chemical class of b. Meclizine
agents? c. Doxylamine
a. Alkylamines d. Pyrilamine
e. Cyproheptadine d. II and III
219. What is the primary mechanism of action of the drug Ondansetron and Granisetron? e. I, II, and III
a. Partiaal agonist effect at the 5-HT1A receptor 226. Epoprostenol, a drug that is being used in the management of primary pulmonary hypertension is an
b. Antagonist effect at the 5-HT1B receptor analogue of which eicosanoid?
c. Full agonist effect at the 5-HT2A receptor a. PGD2
d. Antagonist effect at the 5-HT3 receptor b. PGI2
e. Partial agonist effect at the 5-HT4 receptor c. PGE1
220. What is the clinical use of the drugs Ondansetron and Granisetron? d. PGE2
a. Appetite suppressants e. PGF2-alpha
b. Anti-emetic 227. Which of the following eicosanoids has an analogue that is primary indicated as cytoprotectant in peptic
c. Treatment of acute migraine attack ulcer disease?
d. Migraine prophylaxis
e. Motility enhancing agent a. PGD2
221. What is the clinical use of the drugs that primarily stimulate the 5-HT1B and 5-HT1D receptors? b. PGI2
a. Appetite suppressants c. PGE1
b. Anti-emetic d. PGE2
c. Treatment of acute migraine attack e. PGF2-alpha
d. Migraine prophylaxis 228. Which of the following effects is caused by Prostacylin?
e. Motility enhancing agent a. Bronchoconstriction
222. Sumatriptan is contrindicated in which subset of patients? b. Vasoldilation of the blood vessels
a. Patients with broncospastic disease c. Inhibition of gastric acid secretion
b. Patients with acute attacks of migraine d. Contraction of the uterus
c. Patients with ischemic heart disease e. Inhibition of renin secretion
d. Patients with hypertension 229. What is the primary indication of the prostaglandin analogue Latanoprost?
e. Patients with Wolff-Parkinson-White syndrome a. Management of glaucoma
223. Which of the following statements is/are correct regarding Arachidonic acid? b. Smooth muscle relaxant in erectile dysfunction
I. It is 20-carbon known as eicosapentaenoic acid c. Abortifacient
II. It is mobilized from phospholipids by the enzyme phospholipase A2 d. Inhibition of platelet aggregation
III. It is released from phospholipids by the action of the enzymes phopholipase C and Diglyceride e. Vasoconstrictor in episodes of hypertension
lipase 230. Which of the following statement correctly describes the effects of glucocorticosteroids in eicosanoid
a. I only synthesis?
b. II only I. Dexamethasone inhibits the expression of the COX-2 gene
c. I and III II. Glucocorticosteroids inhibit the synthesis of annexins leading to decreased activity of
d. II and III Phospholipase C
e. I, II, and III III. Glucocorticosteroids stimulates the syntheis of lipocortins which inhibit the activity of
224. Which of the following eicosanoids is a product of the metabolism of Arachidonic acid by the Phospholipase A2
cyclooxygenase enzyme? a. I only
a. Leukotrienes b. II only
b. Isoprostanes c. I and III
c. Epoxides d. II and III
d. Lipoxins e. I, II, and III
e. Prostaglandins 231. Which of the following drugs classified as NSAIDs inhibit/s the COX-1 more than the COX-2 isozymes?
225. Which of the following correctly differentiate/s COX-1 from COX-2 isozymes? a. Indomethacin
I. COX-1 is widely distributed and constitutively expressed b. Ibuprofen
II. COX-1 is expressed upon stimulation by sytokines and growth factors c. Mefenamic acid
III. COX-2 is expressed by inflammatory and immune cells d. Naproxen
a. I only e. Celecoxib
b. II only 232. Which of the following drugs classified as NSAIDs inhibit/s the COX-2 morre than the COX-1 isozymes?
c. I and III a. Indomethacin
b. Ibuprofen a. I only
c. Mefenamic acid b. II only
d. Naproxen c. I and III
e. Celecoxib d. II and III
233. Which of the following statement/s is correctly describe/s the pharmacokinetics of Aspirin? e. I, II, and III
I. Aspirin can be absorbed in the stomach and the small intestines in the hydrolyzed form 239. Which of the following drug is currently considered as the preferred drug to prevent erosive gastritis in
II. Elimination of aspirin can be best descrined as following a Michaelis-Menten kinetics patient at risk of developing this side effect when NSAIDs have to be given to the patient?
III. Acidification of the urine facilitates renal elimination of Salicylates a. Omeprazole
a. I only b. Ranitidine
b. II only c. Sucralfate
c. I and III d. Telenzepine
d. II and III e. Misoprostol
e. I, II, and III 240. In salicylate poisoning, serum salicylate levels of 50-80mg/dL can present with which of the following
234. Which of the following mechanisms may play a role of anti-inflammatory effect of Aspirin? clinical findings?
I. Decreased synthesis of prostaglandins I. Hyperthermia
II. Inhibition of neutrophil migration and adherence II. Respiratpry alkalosis
III. Stabilize iysosomes of neutrophils III. Tinnitus and Vertigo
a. I only a. I only
b. II only b. II only
c. I and III c. I and III
d. II and III d. II and III
e. I, II, and III e. I, II, and III
235. At what usual adult dose of aspirin is its anti-inflammatory effect observed? 241. A25-years old male ingested an unidentified number of Aspirin tablets. He presents with the following
a. 0.3-2.4g/day findings: Responsive to verbal stimulation, BP=110/60, Body Temperature=39.5’C, ABGs: pH=6.9,
b. <0.6-4g/day HCO3=7meq/L, pCO2 -29 mmHg; dry oral mucosa, absent axillary sweating. What serum salicylate level
c. 3.2-4g/day is consistent with these findings?
d. <0.325g/day a. 50-80mg/dL
e. At all dose range b. 80-110mg/dL
236. What is the principal mechanism which can explain anti-pyretic effect of Aspirin? c. 110-160mg/dL
a. Peripheral reduction in the levels of prostaglandins d. 160-190mg/dL
b. Peripheral vasodilation e. >190mg/dL
c. Inhibition of prostaglandin syntheis at subcortical sites 242. A number of the NSAIDs cannot be given to patients with Gouty Arthritis as these may precipitate an
d. Inhibtion of CNS response to interleukin-1 acute attack of gout or induce uric acid stone formation. Which of the following NSAIDs should no be
e. Activation of the thermoregulatory sweat glands given to patient with gout?
237. Which of the following statements is/are correct regarding the anti-platelet aggregation effect of Aspirin? a. Tolmetin
I. Discontinuation of Aspirin immediatel rrestores platelet aggregation in matter of 1-2 days b. Ibuprofen
II. The mechanism involves irreversible actylation of the cyclooxygenase enzyme in platelets c. Indomethacin
III. The effect is clinically significant in all dose ranges for Aspirin d. Diclofenac
a. I only e. Mefenamic acid
b. II only 243. Which of the following NSAIDs with minimal anti-inflammatory activity is/are primarily indicated as
c. I and III analgesic especially in the management of post-operative pain?
d. II and III I. Diclofenac
e. I, II, and III II. Ketorolac
238. Which of the following statements must be considered with regard to the effect of Aspirin aand other III. Etodolac
NSAIDs on the gastric mucosa? a. I only
I. The risk of GI intolerance can be reduced if Aspirin or any other NSAIDs is taken with meals b. II only
II. With Aspirin, the risk of erosive gastritis is obsereved only when Aspirin is given at doses c. I and III
>0.325g/day d. II and III
III. Gastric irritation is most prominent among NSAIDs that inhibit COX-1 more than COX-2 e. I, II, and III
244. All the know NSAIDs are weak acids. Which of the following drugs is the only exception? d. Azathioprine
a. Piroxicam e. Methotrexate
b. Sulindac 251. When the methotrexate is used for the management of rheumatoid arthitis, what is the usual maintenance
c. Nabumetone dosing regimen given to patients?
d. Ketorolac a. 150mg OD
e. Tolmetin b. 15mg OD
245. Which of the following statement/s is/are correct regarding Mefenamic Acid, a popular NSAID? c. 15mg once a week
I. It is more toxic than Aspirin d. 150mg BID
II. The drug should not be used for more than 1 week e. 15mg BID
III. It should not be given to children 252. Which of the following statement/s is/are correct regarding the drug Methotrexate when used for
a. I only rheumatoid arthritis?
b. II only I. The primary mechanism of action involves inhibition of AICAR (aminomidazolecarboxamide)
c. I and III transformylase and thymydylate synthase
d. II and III II. Toxicity of the drug may be minimized with the use of Leucovorin
e. I, II, and III III. The drug is contraindicated in pregnancy
246. What is the most important toxicity associated with pyrazolone derivatives like Phenylbutazone which a. I only
necessitated the withdrawal of a number of these drugs from the market? b. II only
a. Hepatoxicity c. I and III
b. Hematologic toxicities d. II and III
c. Nephrotoxicity e. I, II, and III
d. Gastric ulceration 253. Which of the following statements is/are correct regarding the drug acetaminophen?
e. Stevens-johnson syndrome I. It is a weak inhibitor of COX-1 and COC-2 isozymes in peripheral tissues
247. Which of the following NSAIDs is most useful as an analgesic and has been used successfully to replace II. It has significant anti-inflammatory activity
morphine in some situation involving mild to moderate post surgical pain? When used with an opioid, it III. It is equivalent to Aspirin as analgesic and anti-pyretic
may decrease the opioid requirement by as much as 25-50%. a. I only
a. Naproxen b. II only
b. Ketorolac c. I and III
c. Tenoxicam d. II and III
d. Diclofenac e. I, II, and III
e. Mefenamic acid 254. Which of the following agent is most useful in the management of acetaminophen poisoning?
248. What is the primary advantage od drugs like Celecoxib and etericoxib? a. N-acetylcysteine
a. Less incidence of gastric irritation or ulceration b. S-carboxymethylcysteine
b. Less incidence of nephrotoxicity c. Dantrolene
c. Less incidence of hematologic toxicities d. Atropine
d. Minimal risk of causing worsening hypertension e. Sodium Bicarbonate
e. Less likelihood of inhibiting platelet aggregation 255. Which of the following condition when present may increase of formation of the toxic intermediate N-
249. A number of drugs classified as specificCOX-2 inhibitors like Rofecoxib (Vioxx) have been withdrawn acetyl-p-iminobenzoquinone with regular doses of acteminophen?
from the market or are marketed with “black box” warning. What is the primary reason for such actions to I. Patient with viral hepatitis
be taken against these drugs? II. Patients on warfarin therapy
a. They are associated with severe agranulocytosis III. Chronic alcoholics
b. They cause severe irreversible hepatic necrosis a. I only
c. They increase the risk of bleeding in patient on Warfarin b. II only
d. They increase the risk of thrombosis and cardiac deaths c. I and III
e. They cause worsening of DM and Hypertension d. II and III
250. Which of the following agents is currently used as the first-line DMARDs in the management of e. I, II, and III
Rheumatoid Arthritis? 256. Which of the following drugs used for gout produces its effect by inhibiting the enzyme xanthine oxide
a. Hydroxychloroquine a. Allopurinol
b. Auranofin b. Colchicine
c. Penicillamine c. Indomethacin
d. Probenecid I. Oral bambuterol must be taken on PRN basis for acute exacerbations of bronchial asthma
e. Sulfinpyrazone II. Inhalation of salbutamol for acute relief of attack mest be used for not more than three times a day
257. Which of the following drugs used for gout produces its effect by inhibiting the synthesis of microtubules? III. He must adequately gargle about 5x after each use Fluticasone inhaler
a. Allopurinol a. I only
b. Colchicine b. II only
c. Indomethacin c. I and III
d. Probenecid d. II and III
e. Sulfinpyrazone e. I, II, and III
258. Which of the following is/are appropriate indication/s of Allopurinol? 263. Which of the following drugs for bronchial asthma can be given subcutaneously during acute episodes of
I. In the management of chronic tophaceous gout bronchospasm?
II. In preventing massive uricosuria in patients to be given chemotherapeutic agents for acute a. Salnutamol
leukemia b. Isoproterenol
III. In patients with recurrent renal urate stones c. Terbutaline
a. I only d. Budesonide
b. II only e. Theophylline
c. I and III 264. Which of the following mechanisms may explain the actions of methylxanthines in bronchial asthma?
d. II and III I. May inhibit the late phase allergic reaction occuring about 2-8hours after an acute attack
e. I, II, and III II. May inhibit the enzyme phosphodiesterase leading to an increase in the levels of cAMP
259. In the management of acute gouty attack, which of the following statement/s is/are correct? III. May antagonize the action of Adenosine
I. Uricosuric agent are given at the onset of acute attack to reduce the duration of the episode a. I only
II. Indomethacin is preffered for the management of the pain of an acute attack b. II only
III. Colchine relieves the pain and inflammation of an acute attack in 12-24 hours c. I and III
a. I only d. II and III
b. II only e. I, II, and III
c. I and III 265. Which of the following statements describe/s the clinical use of Theophylline?
d. II and III I. May reduce the nocturnal attack of bronchial asthma when given as sustained-release preparation
e. I, II, and III II. May improve response to high dose inhaledd beta-2 agonists when given intravenously
260. Which of the following groups can relax the brochial smooth muscles and are primarily used as concurrently with beta-2 agonists during acute exacerbation of asthma
bronchodilators in bronchial asthma? III. May improve contratility of the diaphragm improving ventilatory response to hypoxia
I. Methylxanthines a. I only
II. Beta-2 agonists b. II only
III. Anticholinergic agents c. I and III
a. I only d. II and III
b. II only e. I, II, and III
c. I and III 266. Which of the following statements is an important consideration regarding the toxicities and side effects of
d. II and III Theophylline?
e. I, II, and III a. Toxicities and side effects do not occur as long as the serum levels of Theophylline do not exceed
261. In the management of bronchial asthma, which of the following medications is/areconsidered as controller 20mg/L
medications? b. Adults, neonates, and young infants have faster clearance of Theophylline than children
I. Low-dose inhaled glucocorticosteroids c. Clearance may be decreased with inhibition of hepatic enzymes among chronic smokers
II. Short-acting beta-2 agonists d. Seizures and arrythmias are more common when serum levels exceed 40mg/L
III. Long-acting inhaled beta-2 agonists e. Hepatic dysfunction does not alter significantly the clearance of Theophylline and can thus be
a. I only safely used for patients with liver disease
b. II only 267. Which of the following statements describe the use of anti-cholinergic agents in COPD and bronchial
c. I and III asthma?
d. II and III I. They can be safely given at high doses by inhalations to saturate the muscarinic receptors
e. I, II, and III II. Available agents are tertiary ammonium compunds
262. Which of the following advice should you give to a patient with bronchial asthma regarding the use of III. They are more effective than beta-2 agonists in bronchial asthma but less effective in COPD
medications for his condition based on the current recommendations? a. I only
b. II only e. I, II, and III
c. I and III 274. Which of the following is an opioid receptor that is responsible for inducing analgesia most especially
d. II and III among women?
e. I, II, and III a. Kappa
268. What is the mechanism of action of Nedocromil? b. Delta
a. Increase outward potassium conductance leading to hyperpolarization of mast cell membrane c. Alpha
b. Increase inward chloride conductance leading to hyperpolarization of mast cell membrane d. Epsilon
c. Increase inward sodium conductance leading to depolarization of mast cell membrane e. Mu
d. Increase outward calcium conductance leading to repolarization of mast cell membrane 275. Tolerance is an expected consequence with chronic use of most of the available opioids. Which of the
e. Increase inward potassium conductaance leading to depolarization of mast cell membrane following effects of opioids is tolerance NOT observed?
269. What is the mechanism of action of Zafirlukast and Montelukast? a. Respiratory depression
a. Pharmacologic antagonism of Luekotriene D4 action b. Analgesia
b. Pharmacologic antagonism of Leukotriene C4 action c. Euphoria
c. Inhibition of the enzyme 5-lipoxygenase d. Constipation
d. Inhibition of citokine release by macrophages e. Sedation
e. Inhibition of Histamine-1 receptors 276. Which of the following statements regarding the development of tolerance to opioids is/are correct?
270. Which of the following agents has/have experimental effect as bronchodilator/s? I. Develops in 2-3 weeks of continuous use especially when large doses are used over short intervals
I. Omalizumab II. Gross-tolerance develops among predominantly kappa-receptor full agonists
II. Nifedipine III. Partial agonists induce cross-tolerance with strong agonists and antagonists
III. Nitric oxide a. I only
a. I only b. II only
b. II only c. I and III
c. I and III d. II and III
d. II and III e. I, II, and III
e. I, II, and III 277. A 45-year old male admitted for severe chest painwas diagnosed with AMI. He was being given morphine
271. Which of the following correctly describes the action of cromolyn sodium? 2mg IV for relief of the chest pain. Which of the following is/are important findings that must be observed
a. Induces relaxation of bronchial smooth muscles as expected consequence/s of the use of Morphine?
b. Blocks the degranulation of mast cells I. Tachycardia
c. Inhibits release of cytokines by inflammatory cells II. Hypotension
d. Inhibits binding of IgE to mast cell membrane III. Vasospasm
e. Inhibits degradation of cGMP a. I only
272. Which of the following effects observe with acute or chronic use of opioids like morphine can be b. II only
attributed to stimulatio of the mu-receptors? c. I and III
I. Respiratory depression d. II and III
II. Psychosis e. I, II, and III
III. Miosis 278. Which of the following opioids is not associated with biliary colic and thus can be used for severe pain
a. I only associated with this condition?
b. II only a. Morphine
c. I and III b. Pentazocine
d. II and III c. Fentanyl
e. I, II, and III d. Meperidine
273. Which of the following statement correctly describe the mechanism of action of the opioids? e. Codeine
I. They mimic the action of endogenous opioid peptides 279. Abstinence syndrome is a rebound effect characterized by rhinorrhea, lacrimation, yawning, chills,
II. The stimulate the release of endogenous opioid peptides gooseflesh, hyperventilation, and hostility observed in which of the following situations?
III. They bind to receptors like the mu, kappa, and delta I. Abrupt withdrawal of an opioid agonist in a patient taking the drug chronically
a. I only II. In a newborn of a mother illicitly taking heroin during her pregnancy
b. II only III. Administration of Nalbuphine on a patient chronically on morphine
c. I and III a. I only
d. II and III b. II only
c. I and III d. Atropine reduces the absorption and systemic toxicity of Diphenoxylate
d. II and III e. Atropine inhibits the hepatic metabolism of diphenoxylate effectively reducing the dose requuired
e. I, II, and III for diarrhea
280. Which of the following oral doses of morphine is quivalent to 10mg of an IV dose? 287. What is the primary indication for the drug methadone?
a. 200mg a. Analgesic substitute for Morphine
b. 180mg b. Antitussive substitute for Codeine
c. 100mg c. Weaning off patients addicted to Heroin or Morphine
d. 40mg d. Management of biliary colic
e. 10mg e. Anesthetic premedication
281. Diacetylmorphine is the chemical name of which of the following opioids? 288. Which of the following agents is a strong full agonist of opioid receptors
a. Apomorphine a. Hydromorphone
b. Morphine b. Hydrocodone
c. Heroin c. Codeine
d. Codeine d. Propoxyphene
e. Meperidine e. Pentazocine
282. Tramadol is weak mu-receptor agonist that does not require as S2 license for dispensing. It is a synthetic 289. Which of the following agents is classified as a partial agonist of opioid receptors?
analogue ofwhich of the following opiates/opioids? a. Meperidine
a. Codeine b. Methadone
b. Thebaine c. Levallorphan
c. Morphine d. Fentanyl
d. Meperidine e. Nalbuphine
e. Diphenoxylate 290. Which of the following agents is/are classified as opioid antagonist/s?
283. Which of the listed opioid below is along the recommended drugs for the management of acute pulmonary I. Nalorphine
edema? II. Naloxone
a. Nalbuphine III. Naltrexone
b. Codeine a. Meperidine
c. Morphine b. Methadone
d. Meperidine c. Levallorphan
e. Apomorphine d. Fentanyl
284. What is the primary mechanism involved in the utility of opioids in the management of acute pulmonary e. Nalbuphine
edema? 291. Which of the following anticoagulant is produced by recombinant DNA technology from the gene that
a. Induction of osmotic diiuresis of edema fluuids encodes the natural anticoagulant derived from medicinal leeches?
b. Peripheral vasodilation reducing venous return a. Dicumarol
c. Decreased cardiac inotropism b. Hirudin
d. Improved mobilization of edema fluid by osmosis c. Lepirudin
e. Increased glumerolar filtration rate d. Heparin
285. Which of the following is/are possible therapeutic classification/s for available opiates and opioids? e. Dalteparin
I. Cough supressants 292. What is the primary indication of the anticoagulant derived from medicinal leeches or its recombinant
II. Emetics form?
III. Analgesics a. Anticoagulation when necessary during pregnancy
a. I only b. When initiatin anticoagulation therapy in patient requiring chronic anticoagulation
b. II only c. Treatment of and prophylaxis against pulmonary thromboembolism
c. I and III d. Management of thrombosis associated with heparin-induced thrombocytopenia
d. II and III e. In patients with AMI or unstable angina pectoris
e. I, II, and III 293. What is the mechanism of action of regular heparin?
286. What is the main reason why atropine is always given with diphenoxylate (lomotil)? a. Activates antithrombin III which in turn inactivates a number of activated clotting factors
a. The anticholinergic after-effects with Atropine discourages abuse of Diphenoxylate b. Activation of plasminogen to plasmin leading to destruction of fibrin and fibrinogen
b. The anti-motility effect of Atroppine augments the constipating effect of Diphenoxylate c. Inhibits the reductive activation of vitamin K epoxide to the hydroquinone form
c. Atropine minimizes the respiratory depression expected with Diphenoxylate d. Inhibits activvation of prothrombin to thrombin
e. Mimics the action of epsilon-aminocaproic acid 301. When warfarin is used as anticoagulant, the achievement of a maximal hypothrombinemic effect is
294. The activity of regular heparin when given intravenously is best monitored atleast every 6 hours for the expected to be delayed. What is the primary reason for this delay in warfarin effect?
first 24 hours of therapy using what parameter? a. The presence of preformed clotting factors in the plasma which have biiologic half-lives 6 to 60
a. Protime hours
b. Platelet count b. The high degree of protein binding which dela entry of the drug into its biologic site of action
c. Clot reaction time c. The delay of activation of precursor clotting factors due to high levels of activation-inhibitors
d. Activated partial thromboplastin time d. The ability of warfarin to induce its own metabolism which delays achievement of therapeutic
e. Bleeding time levels by days
295. When regular heparin is used subcutaneously, which of the followinh statements is correct? e. The ability of warfarin to interfere with the assay done to measure its activity
a. There is no need to monitor activity in most cases 302. Cutaneous necrosis which can develop during the frist few weeks of therapy is expected with warfarin due
b. Monitoring of activity must be done on a weekly instead of daily basis to which of the following mechanisms?
c. Monitoring must be done using aPTT a. Idiosyncratic and paradoxical increase in the rate of synthesis of a number of clotting factors
d. Monitoring of activity must be done regularly using the PT value b. Earlier decline in the circulating levels of natural anticoagulants causing predominance of
e. Monitoring of activity must be done using the platelet count procoagulant clotting factors
296. Which of the following agents must be classified as a low molecular weight heparin? c. Intrinsic thrombogenic property of warfarin molecules
a. Enoxaparin d. Direct reduction in tissue perfusion as a consequence of the allergic property of warfarin
b. Regular heparin e. Hypotensive effective of warfarin which reditributes blood flow to critical sites in the body
c. Warfarin 303. What is the recommended target protime INR for patients who need warfarin and have prosthetic heart
d. Hirudin valves?
e. Phenindione a. 1-3
297. Which of the following parameters must be measured when a patient on low molecular weight heparin b. 2-3
develops bleeding that may be associated with therapy? c. 3-4
a. aPTT d. 4-5
b. Prothrombin time e. 5-6
c. Factor X assay 304. A patient on warfarin for the last 4 weeks for DVT has a PT INR of 4 but without evidence of bleeding.
d. Factor IX assay What is the most appropriate intervention?
e. Bleeding time a. No intervention as within target INR range
298. Bleeding associated with the use of regular heparin is best managed with which of the following b. Reduction of the dose by atleast half
interventions? c. Adimintration of vitamin K
a. Immediate stopping of therapy d. Adminitration of protamine sulfate
b. Administration of Tranexamic acid e. Administration of tranexamic acid
c. Administration of vitamin K 305. Which of the following drugs or conditions can increase the activity of warfarin by affecting the drug’s
d. Administration of protamine sulfate and stopping therapy absorption, distribution, and elimination?
e. Lowering of the current dose without stopping therapy a. Cimetidine
299. What is the primary drawback with phenprocoumon that limit its clinical use in the management of b. Rifampicin
thrombosis? change to vegetarian diet
a. Ist metabolism to a hepatotoxic metabolite c. The presence of hepatic disease
b. Ita long half-life d. Concurrent use of heparin
c. Its prominent gastrointestinal side effects 306. What interaction is present with concominant use of warfarin and the pyrazolone NSAIDs?
d. Its high degree of protein binding I. Inhibition of urinary excretion of warfarin
e. The long delay in its onset of action II. Inhibition of the oxidative metabolism of S-Warfarin
300. Warfarin exerts its effect primarily by inhibiting the gamma-carboxylation of the glutamate residues of III. Displacement of Warfarin from its protein binding
which of the following clotting factors? a. I only
a. IXa,Xa, XIa, XIIIa b. II only
b. IIa, VIIa, IXa, Xa c. I and III
c. II, VII, IX, X d. II and III
d. IX, X, XI, XIII e. I, II, and III
e. I, II, V, VII 307. Which of the following correctly describes the mechanism of action of fibrinolytic agents?
a. Directly catalyze the proteolysis of fobrin and fibrinogen
b.
Inhibit the action of thrombin by preventing its binding to fibrin and fibrinogen E. Reversible inhibition of endothelial cell cyclooxygenase
c.
Activate plasminogen to plasmin, a serine protease that catalyzes breakdown of fibrin and
313. Which of the following agents requires weekly WBC monitoring within the first 3 months of treatment due to
fibrinogen
a risk of causing leukopenia?
d. Inhibit activation of fibrinogen by proteolysis of thrombin
e. Activate the endogenous anti coagulants protein C and protein S A. Cilostazol C. abciximab
308. Which of the following is classified as a thrombolytic agent?
a. Reteplase B. Dipyridamole D. Clopidogrel
b. Warfarin E.Ticlodipine
c. Argatroban
d. Bivalirudin 314. Which of the following agents is associated with development of thrombotic thrombocytopenic purpura?
e. Lepirudin
A. Aspirin D. Eptifibatide
309. Which of the following statements is/are correct regarding the use of thrombolytic agents in myocardial
infarction? B. Ticlodipine E. Cilostazol

C. Tirobifan
I. Best response is seen if the drug is given within 24 hours from onset of symptoms
315. Which of the following agents is classified as an inhibitor of glycoprotein IIb/IIIa and is useful in patients with
II. Generally given to patients with ST segment elevation
acute coronary syndrome and those undergoing percutaneous coronary intervention?
III. Absolute contraindication is a history of stroke of less than 1 year
A. Clopidogrel D. Cilostazol
a. I only d. II and III
b. II only e. I, II and III B. Ticlodipine E. Urokinase
c. I and III
C. Abciximab
310. Which of the following agents is associated with formation of antibodies that can cause development of
316. What is the most important adverse effect of Dipyridamole when given to patients with ischemic heart disease?
allergic reaction with subsequent exposure to the drug?
A. Significant thrombocytopenia D. Severe diarrhea
a. Urokinase d. Tissue plasminogen Activator
B. Vasospasm of coronary vessels e. intermittent claudication
b. Streptokinase e. Tenecteplase
C. Coronary steal phenomenon
c. Alteplase
317. What is the mechanism of action of the drug Tranexamic acid?
311. Which of the following mechanisms may lead to inhibition of platelet aggregation?
A. ADP inhibition D. Inhibits reductase hydrolysis of Vitamin K epoxide
I. Activation of antithrombin III
B. Serine protease inhibition E. Increase factor VIII activity
II. Thromboxane synthesis inhibition
C. Inhibits plasminogen activation
III. Guanylyl cyclase inhibition
318. Which of the following agents primarily cause lowering of the VLDL and is thus useful for
A. I only d. II and III
hypertiglyceridemia?
B. II only e. I, II and III
C. I and II A. Niacinamide D. Probucol
312. Which of the following mechanisms correctly describes the mechanism of action of aspirin as an inhibitor of B. Fenofibrate E. Cholestyramine
platelet aggregation?
C. Simvastatin
A. Irreversible inhibition of platelet and endothelial cell cyclooxygenase
319. Which of the following is consistent with the action of the drug Colestipol?
B. Irreversible inhibition of endothelial cell cyclooxygenase
I. Prevents reabsorption of bile acids in intestine
C. Irreversible inhibition of platelet cyclooxygenase
II. Increases uptake of LDL and IDL with up-regulation of LDL receptors in the liver
D. Reversible inhibition of platelet and endothelial cell cyclooxygenase
III. Causes an increase in conversion of cholesterol to bile acids
A. I only D. II and III bile acid binding resins E. Methampethamine and other Norepinephrine releasers
B. II only E. I, II and III
C. I and III D. Captopril and other ACE inhibitors

320. What is the mechanism of action of Atorvastatin? 326. Which of the following drugs is associated with cutaneous vasodilation and warmth sensation after an initial
dose which may be reduced with the use of 300mg dose Aspirin taken an hour before the dose?
A. Inhibits the first committed step in the biosynthesis of cholesterol
A. Niacin D. Cholestyramine
B. Serves as ligand for the nuclear transcription called peroxisome proliferators-activated receptor alpha
B. Fenofibrate E. Probucol
C. Stimulates the activity of lipoprotein lipase leading to lipolysis of lipoprotein triglyceride
C. Atorvastatin
D. Inhibits the reuptake of bile acids leading to loss of negative feedback on the alpha-hydroxylase enzyme
327. Which of the following drugs can be bind to molecular components of the macromolecular complex of GABA-
E. Prevents oxidation of LDL cholesterol A receptor –chloride channel present in neuronal membranes in the CNS?

321. Which of the following is/are important monitoring considerations in a patient receivimg Simvastatin? I. Diazepam

I. Baseline and follow-up measurement of serum creatinine II. Zolpidem

II.Baseline and follow-up measurement of amino transferase activity III. Phenobarbital

III. Baseline and follow-up measurement of creatine kinase activity A. I only D. II and III
B. II only E. I, II and III
A. I only D. II and III C. I and III
B. II only E. I, II and III
C. I and III 328. Which of the following statements correctly describes the mechanism of action of the anxiolytic drug
Buspirone?
322. Patients on Statins are generally advised to take the medication at night. What is the basis for this?
A. Selective binding to the BZ-1 receptor in the brain D. Inhibitor of MAO enzyme
A. Maximal absorption occurs at night D. Minimize Syncopal attacks with the first dose of statins
B. Antagonist of alpha receptors in the brain E. Antagonist of the BZ-1 receptor in the brain
B. Maximal cholesterol de-novo synthesis occurs at night E. LDL release occurs at night
C. Partial agonist of 5HT-1A receptor in the CNS
C. Minimize postural hypotension expected with Statins
329. Which of the following benzodiazepines is a prodrug that is hydrolyzed to the active metabolite Nordiazepam
333. Statins are generally best taken at night. Which of the following statins is night dosing unnecessary? in the stomach?

A. Lovastatin D. Pravastatin A. Alprazolam D. Diazepam

B. Simvastatin E. Fluvastatin B. Chlordiazepoxide E. Lorazepam


C. Atorvastatin C. Chlorazepate
324. Which of the following drug combinations is generally avoided?

A. Simvastatin + Niacin D. Gemfibrozil + Colestipol 330. What is the primary use of the drug Flumazenil?

B. Niacin + colestipol E. Simvastatin + Colestipol A. Antagonizes the action of the Benzodiazepines

C. Simvastatin + Gemfibrozil B. Stimulates BZ-1 receptor similar to benzodiazepines.


325. Therapy with HMGCoA reductase inhibitors is associated with Rhabdomyolysis and myositis. These C. Antagonizes the action of Barbiturates
complications which can lead to acute tubular necrosis and renal failure have been reported to be augmented with
co-administration of which drugs. D. Inhibits the 5HT-1A receptor

A. Fenofibrates and other fibrates D. Tranylcypromine and other MAOI E. Stimulates the GABA-A receptor

B. Cholestyramine and other 331. Which of the following Benzodiazepines has the shortest half-life and a rapid onset of action?
A. Lorazepam D. II and III
E. I, II and III
B. Triazolam
336. Which of the following patients may be expected to have an increased sensitivity to sedative-hypnotics leading
C. Oxazepam to higher incidence of adverse effects or toxicity?
D. Diazepam I. Young women

E. Prazepam II. Patients with Cardiovascular disease

332. Which of the following agents is absolutely contraindicated in patients with disorder associated with porphyrin III. Patients with respiratory disease
such as acute intermittent porphyria, vanegate porphyria, and symptomatic porphyria?
A. I only
A. Chlordiazepoxide B. II only
C. I and III
B. Diazepam D. II and III
E. I, II and III
C. Zolpodem
337. Which of the following statements correctly describes the pharmacokinetics of Phenytoin?
D. Phenobarbital
A. Absorption of Phenytoin is constant regardless of the formulation
E. Buspirone
B. Phenytoin is well absorbed after intramuscular injection
333. Which of the following agents is associated with significant anterogade amnestic effect which is advantageous
in a number of medical and surgical procedures that may discomfort to the patient? C. Elimination of Phenytoin follows a first-order kinetics at very low blood levels
A. Midazolam D. A first order kinetics of metabolism is observed with Phenytoin at low and high blood levels
B. Zolpidem E. Phenytoin is not significantly Protein bound so drug displacement interaction is not a major concern
C. Buspirone 338. Which of the following mechanisms explain/s the action of Phenytoin?
D. Phenobarbital I. The drug alters conductance of sodium, potassium, and calcium
E. Thiopental II. At high concentrations, it also inhibits the release of Serotonin and Norepinephrine
334. What is the rationale behind the use of long-acting benzodiazepine like chlordiazepoxide and diazepam in the III. At therapeutic concentrations, Phenytoin inhibits the generation of repetitive action potentials
management of alcohol withdrawal?
A. I only
a. The benzodiazepines produce a paradoxical reaction among chronic alcoholics B. II only
C. I and III
B. Cross-dependence develops among sedative hypnotics D. II and III
E. I, II and III
C. Increase in renal clearance of alcohol
339. Which of the following findings in a patient taking Phenytoin would warrant reduction in the dose?
D. Increase in non-CYP mediated metabolism of alcohol
A. Gingival Hyperplasia
335. Which of the following agents may be expected to have an additive CNS depressants effect with the sedative
hypnotics like benzodiazepines when taken simultaneously? B. Nystagmus
I.Opioid analgesics C. Osteomalacia
II. Phenothiazines D. Ataxia
III. Alcoholic beverages E. Hirsutism
A. I only 340. Autoinduction of the metabolism is a phenomenon that leads to decrease in the usual half-life of a given drug
B. II only that the dose adjustments must be made within the first month of therapy to ensure therapeutic drug levels are
C. I and III
reached, and reduction of dosage must be made once metabolism has stabilized. Such is seen with which of the A. I only
following drugs? B. II only
C. I and III
A. Carbamazepine D. II and III
E. I. II and III
B. Phenytoin
345. What is the most common side effect seen with the use of carbamazepine which requires adjustment in the dose
C. Phenobarbital of the drug?
D. Valproic acid A. Diplopia D. Vomiting
E. Primidone B. Drowsiness E. Leukopenia
341. Which of the following drugs for seizures is metabolized to yield Phenobarbital? C. Hyponatremia
A. Lamotrigine 346. What is the preferred drug for the management of pure absence seizures?
B. Vigabatrin

C. Primidone

D. Topimarate

E. Felbamate

342. When Phenytoin is given intravenously, what is the maximum rate by which drug can be administered?

A. 10mg/min

B. 20mg/min

C. 30mg/min

D. 40mg/min

E. 50mg/min

343. Which of the following statements is/are correct regarding intravenous Phenytoin?
I. Cardiotoxicity is partly due to Propylene Glycol

II. It is compatible with Dextrose containing liquid

III. Fosphenytoin is more soluble and requires no solubilizing agent

A. I only
B. II only
C. I and III
D. II and III
E. I, II and III

344. Which of the following statements is/are correct regarding the toxicity profile and the management of overdose
of most drugs used for treatments of seizures?

I. The most dangerous effect is cardiotoxicity

II. Alkalinization of the urine to hasten elimination is usually effective

III. Stimulants are useful antidotes


A. Phenobarbital III. Minimize nausea and vomiting associated with Levodopa

B. Valproic acid A. I only


B. II only
C. Phenytoin C. I and III
D. II and III
D. Ethosuximide E. I,II and III
E. Lamotrigine 353. What is the mechanism of action of Bromocriptine in the management of Parkinsonism?
347. Which of the following agent is associated with the idiosyncratic hepatotoxicity when given for the treatment of seizure in A. Direct antagonist of D2 receptors
children less than 2 years of age and in patients with multiple medications?
B. Direct agonist of D2 receptors
A. Gabapentin
C. Direct antagonist of 5HT3 receptors
B. Valproic acid
D. Direct agonist of 5HT3 receptors
C. Phenytoin
E. Metabolized in the CNS to Dopamine
D. Primidone
354. Female patients with Pituitary tumor and manifesting with amenorrhea and galactorrhea are initially manage with Bromocriptine.
E. Topimarate What is the mechanism of Bromocriptine in this condition?
348. Which of the following agents is currently the preferred drug for the initial management of Status Epilepticus? A. It inhibits the action Dopamine
A. Lorazepam B. It inhibits the action of Oxytocin
B. Diazepam C. It stimulates the release of Estrogen
C. Phenobarbital D. It inhibits the action of Prolactin
D. Phenytoin E. It stimulates the release of FSH and LH
E. Valproic acid 355. During the maintenance therapy with Levodopa, patients are advsed to take the drug on an empty stomach. Why is this advice
given?
349. Which of the following drugs used or seizure disorders is most commonly associated with cosmetic changes making the drug
desirable to used among adolescents, school-age children, and women? A. To minimize the risk of aspiration in as much as the drug can cause nausea and vomiting
A. Lamotrigine B. Oral bioavailability of the drug is reduced as Levodopa can be found to food particles in the GIT
B. gabapentin C. Bile acids released in response to food can interfere with the absorption of the drug
C. Phenobarbital D. Large neutral amino acids in food interfere with the drugs’ transport acroos the blood brain barrier
E. Food can induce expression of DOPA decarboxylase which can limit the drugs CNS bioavailability
D. Phenytoin
356. Which of the following drugs is most useful in controlling the bradykinesia of Parkinsonism?
E. Valproic acid
A. Levodopa
350. A 50 year old male with Generalized Tonic-clonic seizure was initially given Carbamazepine as maintenance therapy. He however
developed adverse response to drug in form of Stevens-Johnsons syndrome. Which of the following agents may be safed to used as an B. Bromocriptine
alternative treatment for the seizure?
c. Amantadine
I. Valproic acid
D. Selegiline
II. Phenytoin
E. Entacapone
III. Phenobarbital
357. Retroperitoneal fibrosis is an observed complication seen with drugs that belong to what class of agents?
A. I only
B. II only A. Antiviral agents
C. I and III
D. II and III B. Tropane Alkaloids
E. I, II and III
C. Ergot alkaloids
351. Which of the following drugs classified as diuretics maybe useful in epileptic women who have seizures exacerbations at the time
of menses? D. Ethanolamine Anti-histamines

A. Hydrochlorothiazide E. Dopamine Agonist

B. Metolazone 358. Livedo Reticularis, a vascular cutaneous reaction characterized by a reddish-purple, fish net- patterm mottling of the Upper/Lower
extremities, is a side effect associated with which of the following agents?
C. Acetazolamide
A. amantadine
D. Chlorthalidone
B. Di[henhydramine
E. Torsemide
C. Levodopa
352. Which of the following correctly describes the clinical used of Carbidopa in the therapy of parkinsonism?
D. Pramipexole
I. Increased the CNS bioavailability of Levodopa
E. Selegiline
II. Improve symptoms of bradykinesia
359. What is Entacapone?
A. A selective MAO-A inhibitor A. Haloperidol + Thiamylal + Halothane

B. A selective MAO-B inhibitor B. Droperidol + Fentanyl + Nitrous Oxide

C A COMT inhibitor C. Thioridazine + succinyl choline + propofol

D. A Dopamine-3 receptor Agonist D. Nitroue Oxide + Droperidol +Fentanyl

E. A Dopamine-2 receptor agonist E. Halothane + Drperidol + fentanyl

360. The so-called “wearing-off” phenomenon encountered with the use of Levodopa-Carbidopa combination in Parkinsonism is partly 367. Which of the following statement is correct regarding the use of a combination of the least potent Nitrous oxide with a second
due to the formation of which metabolite which can compete with the transport of Levodopa across the blood brain barrier? inhalation anesthetic?

A. alpha-methyldopamine I. Reduction in the risk of the myocardial depression at a given depth of anesthesia compared to the more potent anesthetic given alone

B. alpha-methylnorepinephrine II. Facilitation of metabolism of the more potent anesthetic which may increase the risk of nephrotoxicity

C. alpha-methyldopa III. Reduction in the anesthetic requirement fot he more potane anesthetic due to a second gas effect.

D. 3-O – methyldopa A. I only


B. II only
E. 3-0- methyldopamine C. I and III
D. II and III
361.Which of the currently available inhalation anesthetic is the most potent based in the minimum Alveolar concentration? E. I. II and III
A. Halothane 368. What is responsible for the very short duration and very rapid onset of effect of the so-called Ultra short acting barbiturates?
B. Desflurane A. The presence of a Sulfur group
C. Nitrous Oxide B. The presence of Methoxy group
D. Sevoflurane C. The presence of substituted barbital structure
E. Enflurane D. The high lipid solubility of these agents
362. Based on recent studies what is currently considered as the most probable explanation for the action inhalation anesthetic? E. The increased in the number of polar substituents
A. Interaction with the molecular components of the GABA-A receptor
369. which of the following intravenous agents can only be used to induced anesthesias but not to maintain anesthesia?
B. Interaction and interference with the activity of the fast inward sodium channels
I. Propofol
C. Non-specific interaction with the lipid matrix of the nerve membrane
II. Thiopental
D. Inhibition of the interaction with Glutamate of the NMDA receptor
III. Ketamine
E. Inactivation of the Ligand-gated Potassium, channels
A. I only
363. Which of the following inhalational anesthetics sensitizes the myocardium to cathecolamines which can lead to ventricular B. II only
arryhtmia when symphatomimetic drugs are co-currently administered? C. I and III
D. II and III
A. Sevoflurane E. I, II and III
B. Isoflurane 370. Which of the following anesthetic is associated with a Dissociative state characterized by catatonia, amnesia, and analgesia with or
without actual loss of consciousness?
C. Desflurane]
A. Propofol
D. Halothane
B. Thiopental
E. Nitrous Oxide
C. Midazolam
364. Which of the following inhalational anesthetics associated with potentially severe and Life-threatening hepatitis?
A. Sevoflurane D. Ketamine
B Isoflurane E. Nitrous Oxide
C. Desflurane 371. Which of the following characteristics is consistent with the drug Propofol?
D. Halothane I. Causes a marked reduction in blood pressure during induction
E. Nitrous Oxide II. Recovery from anesthesia is rapid
365. Malignant Hyperthermia which may developed in susceptible individuals exposed to i9nhalational anesthetic is appropriately III. Post-operative nausea and vomiting is minimal due to its anti-emetic effect
treated with which of the following agents?
A. I only
A. Diazepam B. II only
C. I and III
B. Baclofen
D. II and III
C. Dantrolene E. I, II and III

D, Thiamylal 372. What is the common mechanism of action of the Local anesthetics when given in their usual doses?

E. Haloperidol A. Inhibition of Ligand gated sodium channel

366. Which of the following drugs combination provide a state called Neurolepanesthesia? B. Inhibition of the voltage gated sodium channel
C. Stimulate outward potassium flux A. Procaine

D. Stimulate calcium extrusion from the cell B. Lidocaine

E. Inhibit the sodium, potassium ATPase Pump C. Cocaine

373. Which of the following Local anesthetics is hydrolyzed to Para-amino benzoic acid? D. Tetracaine

A. Cocaine E. Prilocaine

B. Isobucaine 380. Which of the following metabolites of local anesthetics when formed in the body in significant amounts can cause oxidation of the
iron in hemoglobin in the ferric form?
C. Lidocaine
A. Glycinexylidide
D. Bupivacaine
B. Ortho-toluidine
E. Tetracaine
C. para amino benzoic acid
374. Allergic reaction is most likely to occur with which of the following agents?
D. Benzoic acid
A. Isobucaine
E. Demethylamine
B. Prilocaine
381. Which of the following anti-psychotics drug groups primarily inhibit the Dopamine-2 receptors in the brain?
C. Procaine
I. Phenothiazines
D. Lidocaine
II. Dibenzodiazepines
E. Mepivacaine
III. Butyrophenones
375. Which of the following local anesthetics is a substrate for metabolic degradation by the enzyme butyrylcholinesterase?
A. I only
A. Chlorprocaine B. II only
C. I and III
B. Etidocaine D. II and III
E. I, II and III
C. Ropivacaine
382. In terms of potency, which of the drugs listed below is expected to be the most potent?
D. Lidocaine
A. Trifluorophenazine
E. Bupivacaine
B. Thioridazine
376. Which of the following agents is considered to be the most neurotoxic when given as a spinal anesthetics causing a transient
reticular irritation? C. Piperacetazine
A. Procaine D. trifluopromazine
B. Lidocaine E. Chlorpromazine
C. Bupivacaine 383. The Anti-psychotic drug Clozapine belongs to what chemical class of agents?
D. Mepivacaine A. Dibenzoxazepine
E.Prilocaine B. Dibenzodiazepine
377. Cardiotoxocity manifesting as a electromechanical dissociation, cardiovascular collapse and death, is a complication of therapy C. Benzisoxazole
most commonly seen with which of the following local anesthetics?
D. Fluorobenzyindole
A. Mepivacaine
E. Thienobenzodiazepine
B. Ropivacaine
384. Which of the following agents is expected to produce the effects and side effects comparable to Haloperidol?
C. Lidocaine
a. Mesoridazine
D. Procaine
B. Chlorprothixene
E. Bupivacaine
C. Thiothixene
378. When large doses of a local anesthetic must be administered, the risk of developing seizures can be significantly reduced with the
used of which of the following agents as pre-medication? D. Promazine

A. Midazolam E. Fluphenazine

B. Thiopental 385. Anticholinergic effect manifesting as loss of accommodation, difficulty in urination, constipation, and a toxic confusional state is
most commonly observed with which of the following drugs?
C. Succinyl choline
A. Haloperidol
D. Propofol
B. Sertindole
E. Diphenhydramine
C. Chlopromazine
379.Which of the following Local anesthetics can predisposed to the development of methemoglobinemia when given in large doses
during regional anesthesia? D. Loxapine
E. Chlorpropthixene II. Relative cholinergic deficiency

386. Dystoinic reactions such as an oculogyric crisis and pseudoparkinsonism such as tremors are side-effects most likely to be seen in III. Dopamine receptor hypersensitivity
patients taking which of the following medications?
A. I only
A. Thioridazine B. II only
C. I and III
B. Chlorpromazine D. II and III
E. I, II and III
C. Loxapine
393. Which of the following drugs is most commonly associated with seizures?
D. Trifluphenazine
A. Thioridazine
E. Molindone
B. Chlorpromazine
387. Neuroleptic malignant syndrome which resembles malignant hyperthermia is more commonly seen with which of the following
agents? C. Loxapine

A. Mesoridazine D. Trifluphenazine

B. Promazine E. Molindone
C. Loxapine 394. Poisonings with antipsychotics are rarely fatal. Which of the following agents is associated with a higher incidence of fatal
poisoning?
D. Haloperidol
A. Mesoridazine
E. Sertindole
B. Chlorpromazine
388. Which of the following drugs for psychosis is most commonly associated with prolongation of QT interval on ECG which can
lead to fatal arrhythmias? C. Trifluphenazine

A. Thioridazine D. Haloperidol

B. Mesoridazine E. Molindone

C. Haloperidol 395. What is the most common side effect associated with the used of tricyclic antidepressants?

D. Clozapine A. Sedation
E. Loxapine B. Tachycardia
389. Which of the following drugs when given to patients will necessitate weekly WBC monitoring for the first 6 months of therapy C. Weight gain
and every 3 weeks thereafter due to its propensity to cause agranulocytosis?
D. Orthostatic hypotension
A. thioridazine
E. Anticholinergic effect
B.Chlopromazine
396. Which of the following agents is classified as a tricyclic antidepressant or 1 st generation anti-depressants?
C. Loxapine
A. Bupropion
D. Clozapine
B Trazodone
E. Molindone
C.Desipramine
390. Which of the following agents is most likely to cause deposits in the retina which can lead to blindness necessitating a baseline
and regular ophthalmologic examination when administered? D. Amoxapine

A. Thioridazine E. Mirtazepine

B. Chlorpromazine
397.Which of the following is/are advantage/s of the SSRI’s compared to the rest of the drugs used for depression?
C. Loxapine I. Lesser lethality with overdose
D. Trifluphenzaine II. Generally no dose titration is required
E. Molindone III. Minimal anticholinergic effects
391. Weight gain is expected complication of therapy with all the clinically useful antipsychotics. One of the drugs listed below, A. I only
however does not produce this side effect. What is this drug? B. II only
C. I and III
A. Thioridazine
D. II and III
B. Chlorpromazine E. I, II and III

C. Loxapine 398. A psychiatric patient on medications develops tremor, thyroid enlargement, and leukocytosis. The drug he is taking is most likely

D. Trifluphenazine A. Chlorpromazine

E. Molindone B. Lithium

392. Tardive dyskinesia is considered as the most important unwanted effect of antipsychotic agents. What is considered as the C. Desipramine
mechanism of development of this condition?
D. Fluoxetine
I. Dopamine receptor block
E. Haloperidol C. I and III
D. II and III
399. Serotonin syndrome which resembles malignant hyperthermia in symptomatology occurs when SSRI’s like Fluoxetine and E. I, II and III
Sertraline are given concurrently with
405. Which of the following agents are avoided in patients taking Tranylcypromine?
I. TCA’s like Nortriptyline
I. Ephedrine
II. MAOI’s like Phenelzine
II. Phenylpropanolamine
III. Heterocyclics like Maprotiline
III. Levodopa
A. I only
B. II only A. I only
C. I and III B. II only
D. II and III C. I and III
E. I, II and III D. II and III
E. I, II and III
400. MAO-A is the amine oxidase primarily responsible for the metabolism of Norepinephrine, Serotonin, and tyramine, while MAO-B
is the specific for the metabolism of Dopamine. Which of the following agents is a selective reversible MAO-A inhibitor whose 406. Which of the following agents can produce the so-called serotonin syndrome when given to a patient already on Phenelzine?
advantage is a lesser risk of causing hypertensive crisis with tyramine foods?
I. Venlafaxine
A. Selegiline
II. Buspirone
B. Nefazodone
III. Fluoxetine
C. Moclobemide
A. I only
D. Trazodone B. II only
C. I and III
E. Imipramine D. II and III
E. I, II and III
401. Which of the following conditions enhance the toxicity of lithium?
407.Which of the following side-effects of Lithium can be seen even in usual doses thus by itself is NOT considered a sign of Lithium
I. Use of thiazide diuretics toxicity ?
II. Diarrhea and vomiting A. Coarse hand tremors
III. Excessive sweating B. Polyuria
A. I only C. hyperreflexia
B. II only
C. I and III D. Confusion
D. II and III
E. I, II and III E. ataxia

402. Imipramine has been found to be useful in the treatment of nocturnal enuresis ( bed wetting). This clinical use explained by which 408. Which of the following may be considered as mechanism/s of effect of Beta lactam antibiotics?
of the following effects of the drug?
I. Inhibition of activity of the transpeptidase enzyme
A. Inhibition of reuptake of norepinephrine
II. Inhibition of formation of the 50s ribosomal subunit
B. Inhibition reuptake of serotonin
III. Binding to the so-called PBPs and related proteins in the cell membrane
C. Anticholinergic effects
A. I only
D. Sedating effect B. II only
C. I and III
E. Inhibition of reuptake of Dopamine D. II and III
E. I, II and III
403. Which of the following findings is/are consistent with the poisoning of antipsychotic drugs?
409. Which of the following statements is/are correct regarding Penicillins?
I. Hypotension
I Penicillins are not metabolized extensively by liver enzyme
II. Hypothermia
II. Probenecid increases Penicillin excretion by blocking renal tubule reabsorption
III. Ventricular arrythmias
III. Ampicillin undergoes enterohepatic cycling
A. I only
B. II only A. I only
C. I and III B. II only
D. II and III C. I and III
E. I, II and III D. II and III
E. I, II and III
404. Which of the following agents interventions is/are appropriate in the management of poisoning with the Phenothiazine
antipsychotic? 410. Which of the following statements in Sulfonamides is/are correct?

I. Gastric lavage is unnecessary one hour after ingestion of tablets I. Their effect is attributed to their ability to inhibit the enzyme dihydrofolate reductase

II. Activated charcoal is given followed by saline cathartics II. As sine agents, they are bacteriostatic but are bactericidal when given with Trimethoprim

III. Epinephrine is the preferred pressor agent for persistent hypertension III. They can cause hemolytic anemia if given to G6PD deficient individuals
A. I only A. I only
B. II only B. II only
C. I and III B. Metronidazole
D. II and III
E. I, II and III C. Chloramphenicol

411. Which of the following may be an expected complication with the use of Aminoglycosides? E. Doxycycline

I. Neuromuscular blockade 418. Which of the following drugs attacks plasmodiain exoerythrocytic stages and is an effective prophylactic agent against
Plasmodium vivax and Plasmodium ovale?
II. Thrombotic Thrombocytopenic purpura
A. Quinine
III. Inhibition of the CYP3A4 enzyme
B. Chloroquine
A. I only
B. II only C. Primaquine
C. I and III
D. II and III D. Sulfadoxine
E. I, II and III
E. Pyrimethamine
412. Which of the following agents is most effective against nosocomial infection with Pseudomonas aeruginosa?
419.Which of the following agents is bactericidal in its activity?
A. Clarithromycin
A. Clindamycin
B. Ceftazidime
B. Erythromycin
C. Amikacin
C. Chloramphenicol
D. Imipenem
D. Doxycycline
E. Metronidazole
E. Ciprofloxacin
413. Which of the following drugs can be given orally to treat infections caused by Peudomonas aeruginosa?
420.Which of the following agents exhibit a primary concentration dependent bactericidal activity wherein the rate and extent of killing
A. Ciprofloxacin increases with increasing concentrations of drug?

B. Ceftazidime A. Amikacin

C. Piperacillin B. Vancomycin

D. Clindamycin C. Clindamycin

E. Azithromycin D. Ceftriaxone

414. Which of the following agents can be safely used for a pregnant women with urinary tract infection? E. Piperacillin

A. Cotrimixazole 421.Among patients with renal impairment, the dose of antibiotics must be adjusted according to the creatinine clearance. Which of the
following drugs does not require such adjustment in dosing for patients with decreased creatinine clearance?
B. Ampicillin
A. Imipenem
C. Norfloxacin
B. Penicillin G
D. Doxycycline
C. Nafcillin
E. Chloramphenicol
D. Ceftazidime
415. Which of the following statements correctly describes the properties of Aminoglycosides?
E. Cefuroxime
I. They are mainly bacteriostatic
422. Which of the following agents requires adjustment or may even have to be avoided in patients with hepatic impairment?
II. They are mainly effective against gram-negative aerobic bacteria
I. Erythromycin
III. They inhibit protein synthesis
II. Metronidazole
A. I only
B. II only III. Clindamycin
C. I and III
D. II and III A. I only
E. I, II and III B. II only
C. I and III
416.Which of the following agents can penetrate inflamed meninges in sufficient concentration to be usedful for meningitis? D. II and III
E. I, II and III
A. Amikacin
423. Which of the following agents is contraindicated in patients less than 18 years of age and pregnant patients due to its adverse
B. Tobramycin effects on cartilage development?

C. Chloramphenicol A. Sulfamethoxazole

D. Aztreonam B. Ethambutol

E. Gentamicin C. Chloramphenicol

417. Which of the following agents has acivity against anaerobic organisms against most protozoan infection? D. Doxycycline

A. Clindamycin E. Ciprofloxacin
424. Which of the following antibiotics is contraindicated in children less than 9 years of age and in pregnant patients due to the risk of E. Oxacillin
permanent staining of teeth and enamel dysplasia in children?
431. Which of the following organism is/are susceptible to Penicillin G, making the drug the drug of choice in the treatment of the
A. Sulfamethoxazole infection caused by this/these organism?

B. Ethambutol I. Treponema pallidum

C. Chloramphenicol II.Pneumococcus

D. Doxycycline III. Beta-hemolytic streptococci

E. Ciprofloxacin A. I only
B. II only
425.Which of the following agents may predispose the neonate premature babies to kernicterus when given to this subset of patients? C. I and III
D. II and III
A. Sulfamethoxazole E. I, II and III
B. Ethambutol 432. The so-called Isoxazolyl penicillins are primarily indicated for the treatment of infection caused by what organism?
C. Chloramphenicol A. Staphylococcus aureus
D. Doxycycline B. Bacteroides fragilis
E. Ciprofloxacin C. Escherichia coli
426. What is the primary role of agents such as Tazobactam amd Sulbactam in antibacterial therapy? D. Haemophilus influenzae
A. Increase in the oral bioavavilability of the Penicillins E. Streptococcus peumoniae
B. Effective against most gram-negative aerobic bacteria 433. Penicillins classified as ureido penicillins have extended spectrum of activity against gram-negative bacteria compared to the
natural and other semisynthetic penicillins. Which of the following organism are the ureido penicillins effective against?
C. Effective against anaerobic bacteria including Clostridia
I. Enterococci
D. Minimize destruction of the Penicillins by bacterial beta-lactamases
II. Klebsiella pneumoniae
E. Improve CNS penetrability of Penicillins
III. Pseudomonas aeruginosa
427.Which of the following antibiotic combinations is associated with clinically significant antimicrobial antagonism which has been
reported to cause higher rate of mortality in patients given this combination compared to patients receiving any of the agent singly? A. I only
B. II only
A. Penicillins + Cephalosporin C. I and III
D. II and III
B. Penicillin + aminoglycoside
E. I, II and III
C. Erythromycin + aminoglycoside
434. Which of the following antibiotics belong to the ureido penicillin class of agents?
D. Penicillin + Chloramphenicol
A. Carbenicillin
E. Erythromycin + Doxycycline
B. Ticarcillin
428.Which of the following is the preferred empiric treatment for patients suspected or proved to have sysytemic fungal infection of
C. Peparicillin
any etiology?
D. Bacampicillin
A. Itraconazole
E. Pivampicillin
B. Amphotericin B
435. Which of the following agents is a first generation cephalosporin that is given intravenously?
C. Fluconazole
A. Cefazolin
D. Flucytosine
B. Cephalexin
E. Ketoconazole
C. Cephadrine
429. The activity of Penicillin G is defined units. How many units does each milligram of crystalline sodium penicillin G contain?
D. Cefadroxil
A. 1
E. Cephapirin
B. 58
436. The second generation cephalosporin includes the Cephamycins. What is the distinctive advantage of the cephamycin over the rest
C. 180
of the second generation cephalosporin?
D. 1600
A. they have greater selectivity against Staphylococcus aureus
E. 2600
B. They have average for anaerobic bacteria
430. The absorption of most oral penicillins is impaired by food. An exception to this is:
C. They are effective against MRSAs
A. Ampicillin
D. They have greater spectrum of activity including Peudomonas aeruginosa
B. Amoxicillin
E. They can be given both orally and parenterally
C. Penicillin
437. Which of the following second generation cephalosporins can be given orally and parenterally?
D. Penicillin G
A. Cefaclor D. Amoxicillin

B. Cefuroxime axetil E. Trimethoprim-Sulfamethoxazole

C. Cefotetan 444. Which of the following Beta-Lactam antibiotic is called a monobactam?

D. Cefprozil A. Moxolactam

E. Cefoxitin B. Aztreonam

438. Which of the following agents belong the cephalosporins group called cephamycins? C. Meropenem

A. Cefamandole D. Loracarbef

B. Cefprozil E. Ticarcillin

C. Cefoxitin 445. What is the use of Cilastatin?

D. Loracarbef A. Effective agent against Pseudomonas aeruginosa

E. Cefaclor B. It prevents tubular secretion of Imipenem

439. Which of the following third generation cephalosporins have activity against Pseudomonas aeruginosa? C. It prevents glomerula filtration of Imipenem

I. Ceftriaxone D. It inhibits the renal dehydropeptidase which metabolizes Imipenem

II. Ceftazidime E. It inhibits liver dehydropeptidase which metabolizes Imipenem

III. Cefoperazone 446. Which of the following antibacterial agents primarily work by inhibiting cell wall synthesis?

A. I only I.Polymyxin B
B. II only
C. I and III II. Vancomycin
D. II and III
E. I, II and III III. Streptpgramins

440. Which of the following agents is considered as fourth generation cephalosporin? A. I only
B. II only
A. Cefepime C. I and III
D. II and III
B. Cefotaxime E. I, II and III

C. Cefoperazone 447. Which of the following agents primarily work by inhibiting aminoacyl translocation and blocking the formation of initiation
complexes?
D. Cefixime
A. Aminoglycosides
E. Moxolactam
B. Chloramphenicol
441. Which of the following cephalosporin is/are associated with Disulfiram-effect when alcohol is taken by patients who are on
this/these antibiotics? C. Linezolid

I. Cefotetan D. Tetracyclines

II. Ceftazidime E. Macrolides

III. Cefamandole 448. Which of the following drugs reversibly binds to the 30S subunit of the bacterial ribosome and inhibits the peptidyl transferase
step of the protein synthesis?
A. I only
B. II only A. Aminoglycosides
C. I and III
D. II and III B. Chloramphenicol
E. I, II and III
C. Linezolid
442. Which of the following cephalosporins is associated with hypoprothrombinema and has increased risk of bleeding disorders?
D. Tetracyclines
A. Cefoperazone
E. Macrolides
B. Cefoxitin
449. Which of the following agents reversibly binds to the 30S subunits of the bacterial ribosome blocking the binding of aminoacyl-
C. Cefuroxime tRNA to the acceptor site on the mRNA-ribosome complex?

D. Ceftriaxone A. Aminoglycosides

E. Cefotaxime B. Chloramphenicol

443. Which of the following agents is considered as first line drug in the management of neisseria gonorrhea infection? C. Linezolid

A. Ceftriaxone D. Tetracyclines

B .Cefuroxime E. Macrolides

C.Penicillin G 450. (MISSING)


451. Which of the following agents inhibits protein synthesis by binding at the 23S ribosomal RNA and preventing the formation of C. Used to eradicate meningococcal carrier state
ribosome complex that initiates protein snthesis?
D. Management of SIADH
A. Aminoglycosides
E. Management of Nephrogenic Diabetes Insipidus
B. Chloramphenicol
458. Which of the following is/are clinical indication/s for the drug Clindmycin?
C. Linezolid
I. Treatment of Severs gram-negative bacterial infection
D. Tetracyclines
II. Treatment of severe anaerobic infections caused by bacteroides
E. Macrolides
III. Prophylaxis of endocarditis in patients with valvular heart disease undergoing dental procedures
452. The mechanism of action of Clindamycin is similar to the mechanism of action of which of the following agents?
A. I only
A. Aminoglycosides B. II only
C. I and III
B. Chloramphenicol D. II and III
E. I, II and III
C. Linezolid
459. Which of the following may be considered as indication for the streptogramin combination Quinupristin-Dalfopristin?
D. Tetracyclines
I. Treatment of MRSA
E. Macrolides
II. Treatment of Multi-Drug resistant strains of Streptococci
453. Which of the following statements correctly describes the properties of the drugs Minocycline and Doxycycline?
III. Treatment of Penicillin-resistant strain of Streptococcus pneumonia
I. They have the longest half-lives among the Tetracyclines
A. I only
II. Their absorption is not impaired by divalent cations like calcium and magnesium B. II only
C. I and III
III. Their absorption is not impaired by food D. II and III
E. I, II and III
A. I only
B. II only 460. Which of the following statements correctly describes the characteristics of Aminoglycosides?
C. I and III
D. II and III I. Exhibit synergistic activity with beta-lactam antibiotics
E. I, II and III
II. May be given both orally and parenterally for systemic activity
454. Which of the following tetracyclines is the drug of choice when tetracyclines are needed in a patient with renal insufficiency??
III. Readily penetrate the blood brain barrier reaching adequate concentrations in the CSF
A. Oxytetracycline
A. I only
B. Chlortetracycline B. II only
C. I and III
C. Methacycline D. II and III
E. I, II and III
D. Doxycyclie
461. Which of the following drugs may enhance toxicity of Aminoglycosides if given with the latter concurrently to a patient?
E. Minocycline
I.Penicillin G
455. Which of the following macrolides can inhibit cytochrome P450 enzymes can cause an increase in the serum concentrations of
drugs like theophylline and warfarin? II. Furosemide
I. Erythromycin III. Vancomycin
II. Azithromycin A. I only
B. II only
III. Clarithromycin
C. I and III
A. I only D. II and III
B. II only E. I, II and III
C. I and III
462. Which of the following Aminoglycosides as the most ototoxic in terms of tis side-effect on the cochlear apparatus?
D. II and III
E. I, II and III A. Neomycin
456. What form of erythromycin has the best oral absorption? B. Amikacin
A. Erythromycin base C. Streptomycin
B. Erythromycin ethylsuccinate D. Tobramycin
C. Erythromycin estolate E. Netilmicin
D. Erythromycin lactobionate 463. Which of the following Aminoglycosides is considered as the most vestibulotoxic?
E. Erythromycin stearate A. Neomycin
457. What is the clinical use of the tetracycline drug Demeclocycline? B. Amikacin
A. Prophylaxis against plague C. Streptomycin
B. Antiprotozoal agent D. Tobramycin
E. Netilmicin E. I, II and III

464. Which of the following Aminoglycosides is considered as the most nephrotoxic? 470. A 3 year old boy is diagnosed with primary tuberculosis infection. The appropriate anti-TB regimen for the child may include
which of the following agents?
A. Neomycin
I. Ethambutol
B. Amikacin
II. Rifampicin
C. Streptomycin
III. Isoniazid
D. Tobramycin
A. I only
E. Netilmicin B. II only
C. I and III
D. II and III
E. I, II and III
465. Which of the following disease-causing organism is/are the aminoglycosides most useful either assingle agents or in
combinations? 471. Which of the following drugs given for tuberculosis is metabolized by the liver enzyme N-acetyltransferase whose expression is
genetically determined may vary in different populations?
I. Mycobacterium tuberculosis
A. Isoniazid
II.Mycobacterium avium intracellulare
B. Rifampicin
III. Staphylococcus aureus
C. Ethambutol
A. I only
B. II only D. Pyrazinamide
C. I and III
D. II and III E. Streptomycin
E. I, II and III
472. Which of the following TB drugs is given only during the intensive phase of therapy?
466. Which of the following agents is given with the Sulfonamides for its synergistic effect with the latter or inhibiting folic acid
synthesis in protozoal cells than mammalian or bacterial cells? A. Isoniazid

A. Pyrimethamine B. Rifampicin

B. Trimethoprim C. Ethambutol

C. Silver D. Pyrazinamide

D. Mafenide E. Streptomycin

E. Salicylic acid 473. What is the acceptable dose for isoniazid in the treatment of TB?

467. What is the mechanism of action of quinolone antibacterial agents? A. 1 mg/kg/day

A. Inhibit the incorporation of GABA into Dihydropteroate B.5mg/k/day

B. Inhibit the bacterial enzyme Topoisomerase II C. 10mg/k/day

C. Inhibit the DNA dependent RNA polymerase of bacteria D. 15mg/k/day

D. Inhibit the Transpeptidation process in bacterial cellwall synthesis E.20mg/k/day

E. Inhibit bacterial dihydrofolate reductase 474. What is the dose for rifampicin in the treatment of TB?

468. A patient receiving sulfadiazine 4 grams per day may benefit with co-administration of which of the following agents as A. 1 mg/kg/day
preventive therapy of sulfadiazine-related complications?
B.5mg/k/day
I. Sodium bicarbonate
C. 10mg/k/day
II. Ammonium chloride
D. 15mg/k/day
III Folinic acid
E.20mg/k/day
A. I only
B. II only 475. Which of the following agents may be considred as useful 2 nd line agents for TB?
C. I and III
D. II and III I. Kanamycin
E. I, II and III
II. Amikacin
469. Which of the following drugs given for tuberculosis is/are bactericidal in activity at the usual dose given for TB?
III. Ciprofloxacin
I. Ethambutol
A. I only
II. Rifampicin B. II only
C. I and III
III. Isoniazid D. II and III
E. I, II and III
A. I only
B. II only 476. Which of the following agents given for tuberculosis may provoke an attack of acute gouty arthritis?
C. I and III
D. II and III A. Isoniazid
B. Rifampicin A. I only
B. II only
C. Ethambutol C. I and III
D. II and III
D. Pyrazinamide E. I, II and III
E. Streptomycin 483. Which of the following agents may interfere with the absorption of levothyroxine that patient must be advised on the proper
interval of intake with this/these agents?
477. Which of the following drugs has been historically associated with the development of Creutzfeldt-Jakob disease, a fatal
neurodegenerative disease caused by prions? I. Sucralfate
A. Pancreas derived insulin II. Ferrous sulfate
B. Pituitary derived growth hormone III. Aluminum hydroxide
C. Thyroid hormone extract A. I only
B. II only
D. Recombinant growth hormone C. I and III
D. II and III
E. Cortisol
E. I, II and III
478. Ocreotide is a drug used for the treatment of such conditions as acromegaly, gastrinoma, and glucagonoma. What hormone is
484. A 43 year old female who works in a hospital was admitted for tachycardia, palpitations and weight loss. Her T4 was elevated
ocreotide an analog of?
with suppresses TSH but her thyroid scan showed a normal-sized gland with normal iodine uptake. Which of the following should be
A. Somatostatin suspected?

B. Growth hormone A. Presence of Grave’s disease

C. Prolactin B. Presence of solitary toxic nodule

D. Oxytocin C. Iodine deficiency goiter

E. Dopamine D. Illicit intake of Levothyroxine

479. Which of the following correctly describes the nomal physiologic hormone release? E. Hypothyroidism

I. GnRH is released continuously by the hypothalamus to induce secretion of FSH and LH by the pituitary 485. A patient is receiving 50mg of Propylthiouracil three times a day. If the plan is to shift her medication to methimazole, what dose
of methimazole should you recommended?
II. ACTH release occurs in pulses that peak iin the early morning hours and after meals
A. 5 mg once daily
III. The release of cortisol is constant throughout the day maintaining a level that varies little with meals
B. 15 mg once daily
A. I only
B. II only C. 30 mg once daily
C. I and III
D. 45 mg once daily
D. II and III
E. I, II and III E. 60 mg once daily
480. Which of the following is/are appropriate indications for oxytocin? 486. In monitoring response to therapy with methimazole, which of the following findings taken 3 months after initiating therapy best
indicates good response to therapy?
I. Control of postpartum uterine bleeding
A. a normal TSH
II. Stimulate initial milk production
B. a normal T4
III. Induction of labor
C. a lower than normal T4
A. I only
B. II only D. a lower than normal TSH
C. I and III
D. II and III E. a higher than normal TSH
E. I, II and III
487. Which of the following describes the mechanism of action of the thioamides for the management of hyperthyroidism?
481. What is the primary use of the drug Desmopressin?
I. Block uptake of iodine by the follicular cells
I. Management of nephrogenic Diabetes insipidus
II. Block the peroxidase-catalyzed reactios
II. Management of Central Diabetes Insipidus
III. Block coupling of iodotyrosines
III. Control of gastrointestinal bleeding
A. I only
A. I only B. II only
B. II only C. I and III
C. I and III D. II and III
D. II and III E. I, II and III
E. I, II and III
488. Which of the following interventions may be appropriate to monitor or treat thionamide-associated agranulocytosis?
482. Which of the following statements correctly describes the thyroid hormones?
I. Discontinuing the offending drug usually reverses the side-effect
I. T3 is better absorbed after oral administration than T4
II. Monitoring the WBC is necessary for the first three months of therapy
II. Peripheral deiodination of T3 converts it to more active T4
III. Switching of therapy from one thionamide to another may reverse the effect
III. Hypothyroidism increases the clearance of both hormones leading to an decrease in their half-lives
A. I only C. I and III
B. II only D. II and III
C. I and III E. I, II and III
D. II and III
E. I, II and III 495. Which of the following drugs or drug combinations is appropriate maintenance therapy for patient with primary adrenal
insufficiency?
489. Which of the following cautions must be observed when iodides are given to treat hypothyroidism?
A. Hydrocortisone + Fludrocortisone
I. Iodides must be initiated before thionamide therapy
B. Dexamethasone + Hydrocortisone
II.Iodides must be avoided if treatment with radioactive iodine is likely
C. Hydrocortisone
III. Iodides are the preffered drugs for chronic therapy during pregnancy
D. Prednisone
A. I only
B. II only E. Betamethasone
C. I and III
D. II and III 496. Which of the following is/ are appropriate indications for using Glucocorticosteroids?
E. I, II and III
I. In mothers who may have premature labor and delivery before 34 weeks of gestation
490. What is the role of beta-blockers in the management of thyrotoxicosis?
II. In patients with secondary pulmonary tuberculosis prior to initiation of antitubercular therapy
I. beta-blockers can inhibit the peroxidase-catalyzed reaction in the synthesis of thyroid hormones
III. In patients who have recently undergone organ transplantation
II. Beta-blockers control the sympathetic symptoms of hyperthyroidism
A. I only
III. Beta-blockers may inhibit the peripheral conversion of T4 to T3 B. II only
C. I and III
A. I only D. II and III
B. II only E. I, II and III
C. I and III
D. II and III 497. In patients on glucocorticoid therapy, which of the following monitoring plans sould be initiated?
E. I, II and III
I. Blood pressure monitoring
491. Which of the following statements correctly describes the thyroid hormone replacement therapy?
II. Bone density monitoring
I. Infants and children with congenital hypothyroidism require higher dose per kilogram body weight than adults
III. Blood sugar monitoring
II. Steady state levels of thyroxine takes about 6-8 weeks to achieve after initiating therapy
A. I only
III. Older adults and those with long-standing disease must be started on lower than usual dose of levothyroxine B. II only
C. I and III
A. I only D. II and III
B. II only E. I, II and III
C. I and III
D. II and III 498. Which of the following statements is/are correct regarding adrenal suppression as a complication of therapy with
E. I, II and III glucocorticosteroids?
I. It is expected to occur when glucocorticosteroids therapy is extended beyond 2 weeks
492. A 20-year old female is receiving Hydrocortisone 20 mg IV three times a day. If the plan is to discharge the patient with an
equivalent dose of Oral Prednisone, what dose of Prednisone will you recommend? II. Dosage reduction when the therapy is to stopped must be done very slowly when the dose reaches replacement levels

A. 5 mg once a day III. Patients must receive supplementary therapy at times of stress like surgery or trauma

B. 10mg once a day IV. Recovery of the hypothalamic-pituitary-adrenal axis is rapid after discontinuing chronic use of Dexamethasone

C. 10 mg in the morning and 5 g in the afternoon V. Treatment with ACTH reduces the time required for the return of normal adrenal function

D. 15 mg in the morning and 10 mg in the afternoon A. I and III

E. 20 mg in the morning and 15 mg in the afternoon B. II and IV

493. What is the physiologic dose of glucocorticoid? C. I, II and III

A. Hydrocortisone dose less than 10-20 mg/day D. I, III and V

B. Hydrocortisone dose greater than 10-20mg/day E. I, II ,III, IV and V

C. Prednisone dose ranging from 10-20mg/day 499. Which of the following glucocorticosteroids is employed in the work-up of patients with Cushing’s Syndrome to diagnose the
most probable etiology of the condition?
D. Prednisone dose less than 10-20mg/day
A. Hydrocortisone
E. Prednisone dose less than 10-20mg/day
B. Prednisone
494. Which of the following effects is/are consistent when cortisol in the body is less than 10 mg/day?
C. Cortisone
I. Metabolism of carbohydrate, fat and proteins
D. Fludprednisolone
II. Inhibition of release of cytokines from macrophages
E. Dexamethasone
III. Enhanced vascular and bronchial smooth muscle response to cathecolamiines
500. What is the Primary role if Zinc in commercial insulin preparations?
A. I only
B. II only A. Improve stability and shelf-life of insulin
B. Improve the solubility of insulin D. Maintain his pre-breakfast insulin at 20 units while increase his pre-supper insulin to 12 units

C. Enhance the activity of insulin in receptors E. No change in the insulin doses as all values are withih the acceptable range

D. Facilitate absorption of insulin from subcutaneous tissue 507. Immune insulin resistance is said to be due to production of low levels of what type of insulin antibodies?

E. Enhance the activity of suspending agents A. IgA

501. The transport of glucose that is facilitated by insulin is mediated by which of the following transporters? B. IgD
A. GLUT1
C. IgE
B. GLUT2
D. IgG
C. GLUT3
E. IgM
D. GLUT4
508. Which of the following oral anti-diabetic drugs is/are known to induce hypoglycemia among diabetic and euglycemic individuals?
E. GLUT 5
I. Sulfonylureas
502. What is the primary advantage of Insulin Lispro compared to regular Insulin?
II. Biguanides
A. Longer duration of action reaching up to 6 hours
III. Meglitinides
B. No associated peak levels
A. I only
C. It is less immunogenic B. II only
C. I and III
D. Less likelihood to cause hypoglycemic episodes D. II and III
E. I, II and III
E. Rapid onset of action in 5 to 15 minutes
509. Which of the following antidiabetic drugs produced control of blood sugar by promoting directly or indirectly the release of
503. Which of the following new insulin preparations has a characteristics release pattern that shows no peak and a plateau serum insulin by the remaining functional beta cells of the pancreas?
insulin level that is maintained for about 24 hours?
I. Meglitinides
A. Insulin Glargine
II. Biguanides
B. Insulin Lispro
III. Thiazolidinediones derivatives
C. insulin Aspart
A. I only
D. Ultralente Insulin B. II only
C. I and III
E. NPH Insulin D. II and III
E. I, II and III
504. Which of the following statements is/are correct regarding Lente Insulin?
510. Which of the following antidiabetic drugs has the longest half-life?
I. It is a mixture of 30% semilente and 70% ultralente Insulin
A. Chlorpropramide
II. It is also known as NPH
B. Tolazamide
III. It is an intermediate-acting Insulin preparation
C. Tolbutamide
A. I only
B. II only D. Glyburide
C. I and III
D. II and III E. Glimepiride
E. I, II and III
511. What is the recommended clinical use of Repaglinide in the management of Diabetes mellitus?
505. A 50 year old male patient with Type2 Diabetes mellitus is maintained for the last 7 days on subcutaneous Isophane Insulin at 20
units and 10 units pre-supper. For the last three days, his capillary blood glucose levels have been consistently showing the following A. Primarily given to control fasting blood glucose levels
values: 190 mg% (prebreakfast), 118mg% (2hr-post lunch), and 110 mg% (presupper). What recommendation will you make regarding
his therapy? B. Improves tissue response to insulin

A. Increase his pre-breakfast insulin to 24 units while maintaining his pre-supper Insulin at 10 units C. Control post-prandial hyperglycemia

B. Increase his pre-breakfast Insulin to 24 units and his pre-supper Insulin to 12 units D. Control overnight blood sugar

C. Maintain his pre-breakfast and pre-supper Insulin doses give Insulin Lispro 4 units pre-breakfast E. Prevent absorption of carbohydrates

D. Maintain his pre-breakfast insulin at 20 units while increase his pre-supper insulin to 12 units 512. Which of the following drugs is a Biguanide?

E. No change in the insulin doses as all values are within the acceptable range A. Acetohexamide

506. A 50 year old male patient with type2 diabetes mellitus is maintain for the last 7 days of subcutaneous Isophane Insulin at 20 Units B. Rosiglitazone
pre-breakfast and 10 units pre-supper. For the last three days, his capillary blood glucose levels have been consistently showing the
following values: 110mg% ( pre-breakfast), 218mg% (2 hour post breakfast), 110 mg% (2 hour post lunch), and 110 mg% ( pre- C. Nateglinide
supper). What recommendation will you make regarding his therapy?
D. Metformin
A. Increase his pre-breakfast insulin to 24 units while maintaining his pre-supper insulin at 10 units
E. Acarbose
B. Increase his pre-breakfast insulin to 24 units and his pre-supper insulin to 12 units
513. In which individuals should Biguanides be avoided as part of therapy for DM because of increased risk of lactic acidosis?
C. Maintain his pre-breakfast and pre-supper insulin doses but give insulin lispro 4 units pre-breakfast
I. Chronic alcoholics A. I only
B. II only
II. Coronary Artery disease C. I and III
D. II and III
III. Chronic hepatits E. I, II and III
A. I only 520. Which of the following drugs is recommended prophylaxis for travelers in malaria endemic areas with known chloroquine-
B. II only resistant strains of Plasmodium falciparum?
C. I and III
D. II and III I. Primaquine
E. I, II and III
II. Mefloquine
514. Which of the following drugs is involved in regulating the genes involved in lipid and glucose metabolism and adiposite
differentiation by acting as ligand of the PPAR-gamma, thus useful for the managing Insulin resistance? III. Atovaquone-proguanil

A. Metformin A. I only
B. II only
B. Repaglinitide C. I and III
D. II and III
C. Acarbose E. I, II and III
D. Tolazamide 521. Although the clinical value of resistance reversal is not established, there are certain drugs that have been shown to reverse
chloroquine-resistance in malaria. Which of the following is an exemple of resistance reversing drugs?
E. Rosiglitazone
A. Verapamil
515. Which of the following drugs may be given to type 1 DM patients as a combination therapy with Insulin?
B. Propranolol
A. Voglibose
C. Calcium gluconate
B. Repaglinide
D. Primaquine
C. Nateglinide
E. Diphenhydramine
D. Chlorpropamide
522. Which of the following is the preferred treatment of malaria caused by Plasmodium vivax or Plasmodium ovale?
E. Glyburide
A. Clindamycin
516. Which of the following are clinical use of Glucagon?
B. Doxycycline
I. Reverse the physical deformity associated with acromegaly
C. Fansidar
II. Management of severe hypoglycemia
D. Halofantrine
III. Reverse the cardiac effects of beta blockers overdose
E. Chloroquine
A. I only
B. II only 523. What is the treatment of choice for the management of severe falciparum malaria?
C. I and III
D. II and III A. Chloroquine phosphate
E. I, II and III
B. Quinidine gluconate
517. Which of the following is the common complication associated with the alpha-glucosidase inhibitors?
C. Artemether
A. Cardiac arrhythmia
D. Artesunate
B. Flatulence
E. Doxycycline
C. Congestive heart failure
524. Which of the following agents may be useful for eradicating a carrier state of Entamoeba histolytica?
D. Hepatotoxicity
I. Metronidazole
E. Interstitial Nephritis
II. Diloxanide furoate
518. Which of the following is the most common problem encountered in female patients using Progestin only oral contraceptive pills?
III. Iodoquinol
A. Mastalgia
A. I only
B. Breakthrough bleeding B. II only
C. I and III
C.Increase in pigmentation D. II and III
E. I, II and III
D. Hirsutism
525. Which of the following agents is considered as the standard drug for the initial management of Pneumocystis jiroveci (carinii)
E. hypertension pneumonia in patients with AIDS?
519. The risk of stroke is especially increase when oral contraceptives are given to which subpopulation of patients? A. Cotrimoxazole
I. Age above 35 years old B. Pentamidine
II. Use of progestin only pills C. Doxycycline
III. Chronic smokers D. Meronidazole
E. Emetine A. I only
B. II only
526. filariasis an infection which can lead to Lymphatic obstructive disease is caused by Wuchreria bancrofti and Brugia malayi. C. I and III
Eradication of microfilaria can be accomplished by the use of which of the following agents? D. II and III
E. I, II and III
A. Praziquantel
533. The ability of a chemical agent to cause injury in a given situation or setting is called?
B. Diethycarabamazine citrate
A. Hazard
C. Niclosamide
B. Risk
D. Albendazole
C. Exposure
E. Metronidazole
D. Toxicity
527. What is the probable mechanism of action of Mebendazole, A broad spectrum anthelmintic agent for Ascariasis, Trichuriasis, and
hookworm infection? E. Threshold

A. Inhibition of the incorporation of PABA in to dehydropteroate 534. What is considered as the most common cause of air pollution?

B. Intensifies GABA-mediated transmission in peripheral nerves of nematodes A. Sulfur oxides


C. Inhibition of cholinesterase action in parasites leading to paralysis of the worm B. Nitrogen oxides
D. Inhibition of the microtubule synthesis C. Hydrocarbons

E. Increases permeability of the nematode cell membrane to calcium resulting to paralysis and death D. Carbon monoxide

528. Which of the following agents is considered as the drug of choice for most cestodal and trematodal infection? E. Ozone

A. Praziquantel 535. Which of the following aliphatic hydrocarbons is considered as the most hepatotoxic?

B. Mebendazole A. Carbon monoxide


C. Niclosamide B. Chloroform
D. Albendazole C. Trichloroethylene

E. Metronidazole D. Tetrachloroethylene

529. Which of the following antineoplastic agents is considered as cell cycle specific agent in terms of its action on cancer cells? E. Methyl chloroform

A. Dactinomycin 536. What is the most important toxidrome of acute exposure to benzene?

B. Daunorubicin A. CNS Stimulation

C. Vincristine B. CNS depression


D. Cisplatin C. Bone marrow injury
E. Mechlorethamine D. Peripheral neuropathy
530. Exemestane is a steroidal hormone agent that binds to and irreversibly inactivate aromatase enzyme. What is the clinical use of the E. Leukemia
agent?
537. Which of the following is considered as the most significant serious toxic effect of chronic exposure to benzene?
A. Acute myelogenous leukemia
A. Hepatotoxicity
B. Renal cell carcinoma
B. Nephrotoxicity
C. Choriocarcinoma
C. Peripheral neuropathy
D. Breast cancer
D. Bone marrow injury
E. Colorectal cancer
E. Cardiotoxicity
531. Which of the following correctly provides the sequence of symptoms associated with carbon monoxide poisoning?
538. What is considered as the most common cause of industrial lead poisoning?
A. Psychomotor impairment headache confusion tachycardia coma
A. Absorption of Lead via the gastric mucosa
B. Convulsions headache tachycardia loss of visual acuity coma
B. Absorption of lead via the respiratory tract
C. Tachycardia confusion psychomotor impairment loss of visual acuity coma
C. Absorption of lead through the skin
D. Coma confusion headache tachycardia loss of visual acuity
D. Absorption of lead from the antiknock gasoline
E. Convulsions loss of visual acuity confusion tachycardia coma
E. Absorption of lead from the intestines
532. Which of the following statements regarding carbon monoxide poisoning is/are correct?
539. Which of the following factors predisposed to increased intestinal lead absorption?
I. The brain and the heart are the most affected organ
I. Increased dietary Calcium
II. Forms carboxyhemoglobin which cannot transport oxygen
II. Iron deficiency
III. Smokingmales have a higher carboxyhemoglobin level
III. Ingestion in an empty stomach D. Alters DNA transcription by impairing synthesis of mRNA

A. I only E. Intercalates with the DNA leading to failure of gene expression


B. II only
C. I and III 546. Hepatolenticular degeneration is a result of chronic exposure to which metal?
D. II and III
E. I, II and III A. Lead

540. Which of the following statements is/are correct regarding the pharmacokinetics of lead? B. Mercury

I. Lead can cross the placenta and pose a potential hazard to fetus C. Iron

II. Adults have greater degree of absorption of ingested lead than young children D. Copper

III. The major route of excretion of lead is through the urine E. Arsenic

A. I only 547. Chronic excessive exposure to this metal can lead to deposition in various organs and tissues causing the development of
B. II only conditions such secondary Diabetes mellitus, restrictive cardiomyopathy, and hepatic failure
C. I and III
D. II and III A. Lead
E. I, II and III
B. Mercury
541. In developing a fetus as well as in a young child, what is considered as the most sensitive target organ for Lead’s toxic effect?
C. Iron
A. Active bone marrow
D. Copper
B. Kidneys
E. Arsenic
C. Immature reproductive organs
548. Erethism, a behavioral pattern characterized by change in mood from shyness, withdrawal and depression with explosive anger or
D. Gastrointestinal tract blushing, is seen as a manifestation of metal poisoning. It can be seen with other findings such as tremors progressing to choreiform
movements of limbs and gingivostomatitis. Which of the following metals may be responsible for these manifestations?
E. Developing central nervous system
A. Lead
542. What is the primary repository site of lead in the body of an adult containing about 90% of the total body lead burden?
B. Mercury
A. Liver
C. Iron
B. Bone
D. Copper
C. Brain
E. Arsenic
D. Gastrointestinal tract
549. The findings of congestive cardiomyopathy, pulmonary edema (cardiogenic and non-cardiogenic), pancytopenia with basophilic
E. Skin and its appendages stippling of erythrocytes, and ascending sensorimotor peripheral neuropathy with acute exposure, and the appearance of Aldrich-mees
lines months after the acute exposure are consistent with poisoning with which of the following metals?
543. Which of the following chelating agents may be useful in the different phases of management of inorganic lead poisoning?
A. Lead
I. Calcium disodium edetate
B. Mercury
II. Succimer
C. Iron
III. Dimercaprol
D. Copper
A. I only
B. II only E. Arsenic
C. I and III
D. II and III 550. Chronic exposure with this metal has been linked with malignancies of the lung, skin, and the urinary bladder even at doses not
E. I, II and III high enough to cause either acute or chronic toxicities. Which toxic chronic exposures on the hand, important findings include a
“raindrop” pattern of hyperpigmentation and hyperkeratosis of the palms and soloes,and a stocking-glove pattern of sensorimotor
544. A patient presenting with encephalopathy, abdominal colic, hemolytic anemia and an elevated liver enzymes several days after neuropathy?
exposure to large amounts of a metallic poison may be suffering from what poisoning?
A. Lead
I. Acute inorganic lead poisoning
B. Mercury
II. Acute Inorganic arsenic poisoning
C. Iron
III. Acute mercury
D. Copper
A. I only
B. II only E. Arsenic
C. I and III
551. Which of the following manifestations is most consistent with acute overdose of Iron, particularly in children?
D. II and III
E. I, II and III A. Hemorrhagic gastroenteritis
545. Which of the following effects correctly describe the mechanism of action of arsenic as a poison? B. Acute hepatic necrosis
A. Inhibits generation of ATP by binding to cytochrome oxidase system in the electron transport chain C. Acute pulmonary edema
B. Inhibits effective oxygen delivery by oxidizing the hemoglobin iron form ferrous to ferric D. Encephalopathy and convulsions
C. Inhibits the enzyme activity by binding to the Sulfhydryl groups in the enzyme E. Acute Gingivostomatitis
552. Which of the following is/are correct indications for the use of Dimercaprol? D. Sodium bicarbonate

I. Single-agent treatment of acute poisoning with inorganic or elemental mercury E. Magnesium sulfate

II. Single-agent treatment of acute arsenic poisoning 559. Seizures as a manifestation of poisoning can be seen as a common component of the toxidrome of which of the following drugs?

III. Combination treatment with edentate calcium disodium for severe lead poisoning I. Atropine

A. I only II. Methampethamine


B. II only
C. I and III III. Tricyclic antidepressants
D. II and III
E. I, II and III A. I only
B. II only
553. Penicillamine, as a chelating agent, is primarily used for the treatment of poisoning with which of the following metals? C. I and III
D. II and III
A. Lead E. I, II and III

B. Mercury 560. Which of the following agents can cause a state of drunkenness followed a day later by severe anioin gap metabolic acidosis?

C. Iron A. Ethylene glycol

D. Copper B. Ethanol

E. Arsenic C. Gamma-hydroxybutyrate

554. Deferoxamine is primarily indicated for the treatment of poisoning with which of the following metals? D. Lorazepam

A. Lead E. Diazepam

B. Mercury 561. The volume of Distribution of a drug may determine the efficacy of hemodialysis or hemoperfusion as a means of facilitating
elimination of the drug from the body. Which of the following drugs or drug groups have a high volume of distribution which makes
C. Iron hemodialysis ineffective therapeutic option in cases of poisoning?

D. Copper I. Antipsychotics

E. Arsenic II. Antidepressants

555. Hemodialysis as an intervention for poisoning is indicated for which of the following drugs? III. Antimalarials

A. Morphine A. I only
B. II only
B. Digoxin C. I and III
D. II and III
C. Diazepam E. I, II and III
D. Propranolol 562. Constriction of pupils (miosis) is seen with poisoning or overdoses with which of the following agents?
E. Ethylene glycol A. Opoids
556. The use of activated charcoal as a means of eliminating a drug taken in overdose is applicable for which of the following drugs? B. Tropane alkaloids
A. Carbamazepine C. Amphetamines
B. Ethanol D. LSD
C. Ferrous sulfate E. Cocaine
D. Lithium carbonate 563. Which of the following agents can cause a high anion gapmetabolic acidosis?
E. Potassium chloride I. Methanol
557. Following a non-accidental ingestion of Acetaminophen tablets, what minimum serum acetaminophen level taken 4 hours after II. Ethanol
ingestion will indicate a high risk of liver injury?
III. Metformin
A. > 10mg/ml
A. I only
B. > 50mg/L B. II only
C. I and III
C. > 1000mg/L
D. II and III
D. > 1500 mg/L E. I, II and III

E. >2000mg/L 564. What is the preferred treatment for severe Verapamil overdose?

558. In the management of Quinidine-like cadiotoxicity associated with an overdose of a tricyclic antidepressant, which of the A. Propranolol
following is the most useful intervention?
B. Sodium bicarbonate
A. Physostigmine
C. Epinephrine
B. Lidocaine
D. Calcium gluconate
C. Quinidine
E. Naloxone
565. Which of the following is most appropriate treatment for the management of cardiac manifestations of theophylline or Caffeine I. Patients with Bronchial asthma
overdose?
II. Patients taking warfarin
A. Verapamil
III. Patients with hypertension
B. Esmolol
A. I only
C. Digoxin B. II only
C. I and III
D. Magnesium sulfate D. II and III
E. I, II and III
E. Adenosine
572. Delayed neurotoxicity associated with exposure to organophosphate characterized by polyneuropathy, paralysis and axonal
566. Which of the following agents is most useful for the management of acetaminophen toxicity? degeneration has been attributed to what effect of these toxicants?
A. S-carboxymethylcysteine A. Inhibition of the plasma butyrylcholinesterase
B. N-acetylcysteine B. Inhibition of release of Acetylcholine
C.Atropine C. Inhibition of the RBC acetylcholinesterase
D. Ethanol D. Direct inhibition of myelin sheath function
E. Oxygen E. Inhibition of the neuropathy target esterase
567. Which of the following agents may be useful for the treatment of poisoning due to methanol? 573. Which of the following statements is/are correct regarding the toxicology of the carbamate insecticides?
I .Ethanol I. Inhibition of the cholinesterase enzyme is reversed easily with the use of pralidoxime
II. Glucagon II. The clinical approach to management is similar to organophosphate poisoning
III. Fomepizole III. They are considered as persistent toxicants with great impact in the environment
A. I only A. I only
B. II only B. II only
C. I and III C. I and III
D. II and III D. II and III
E. I, II and III E. I, II and III
568. Which of the following agents may be useful for the treatment of ethylene glycol poisoning? 574. Which of the following mechanisms explains the toxicity of the chlorinated hydrocarbon insecticides like DDT?
I .Ethanol A. inhibit the ligand-gated sodium channels
II. Glucagon B. Inhibit the enzyme acetylcholinesterase
III. Fomepizole C. Stimulate the GABA-A receptor
A. I only D. Inhibit the inactivation of sodium channels
B. II only
C. I and III E. Promote outward potassium transport
D. II and III
E. I, II and III 575.Toxidromes of poisoning with chlorinated hydrocarbons primarily consists of which of the following manifestations?

569. Royal jelly, a popular nutritional supplement form honeybee has been reported to cause which of the following adverse effects? A. CNS stimulation

A. Hepatotoxicity B. CNS depression

B. CNS depression C. Bone marrow suppression

C. CNS stimulation D. Cardiac rhythm disturbance

D. Anaphylaxis E. Hepatic failure

E. Hemorrhagic gastritis 576. Which of the following agents is/are considered as botanical insecticide/s (derived from botanical sources)?

570. Ginkgo biloba is a common additive in a number of nutritional supplements. Which subset of patients should be cautioned against I. Nicotine
the use ginkgo-containing products?
II. Rotenone
I. Patients receiving anticoagulation therapy
III. Pyrethrum
II. Patients with Alzheimers Dementia
A. I only
III. Patients with history of seizures B. II only
C. I and III
A. I only D. II and III
B. II only E. I, II and III
C. I and III
D. II and III 577. What is the minimum fatal dose of nicotine, equivalent to 2 sticks of cigarettes?
E. I, II and III
A. 10mg
571. Ginseng (Panax ginseng) is a popular additive in nutritional supplement due to its ability to improve physical and mental
performance. Which subset/s of patients must be advised regarding the potential adverse effect of Ginseng on their disease or when B. 20 mg
taken concurrently with their medications?
C. 40mg 584.Yusho disease documented in Japan in the 1960’s is associated with exposure to which of the following toxicants?

D. 80 mg A.Polychlorinated biphenyls

E. 160 mg B. Paraquat

578. Which of the following findings is/are consistent with acute nicotine toxicity? C. Nicotine

I. Hypertension D. Aconite

II. Peripheral vasodilatation E. Mercury

III. Cardiac arrhythmia 585. What is considered as the most common mode of exposure to poisons?

A. I only A. inhalation
B. II only
C. I and III B. Dermal exposure
D. II and III
E. I, II and III C. Ingestion

579. Which of the following describe/s the appropriate management of acute nicotine toxicity? D. Ophthalmic

I. Anticonvulsant therapy mainly involves the use of benzodiazepine like Diazepam E. Bites

II. Epinephrine is given to increase the blood pressure 586. In the initial treatment of poisoning at home, which of the following instructions is correct to give regarding inducing vomiting as
an initial intervention?
III. Neostigmine is given to counteract the neuromuscular blockade
A. Vomiting can be induced safely with either Extract of Ipecac syrup
A. I only
B. II only B. for adults, use 2 tablespoonfuls of Ipecac syrup followed by a glassful of water
C. I and III
D. II and III C. For children, less than 1 year old, use one tablespoonful of Ipecac followed by a glassful of water
E. I, II and III
D. If the patient does not vomit with Ipecac, use salt water or mustard as an alternative
580. Paraquat is described as having a toxicity rating of 4. What are the probable human lethal dosages with this rating?
E. If the patient does not vomit with Ipecac, try other techniques like tickling the back of the throat of your finger
A. 5,000-50,000mg/kg body weight
587. Which of the following situations will contraindicate induction of vomiting as an intervention in cases of poisoning?
B. 500- 5,000mg/kg body wt
I. Patient has ingested petroleum products
C. 50- 500mg/kg body wt
II. Patient is less than 1 year old
D. 5-50mg/kg body wt
III. Patient is having convulsion
E. not more than 5 mg/kg body wt
A. I only
581. Parathion has a toxicity rating of 6. What are the probable human lethal dosages with this rating? B. II only
C. I and III
A. 5,000-50,000mg/kg body weight D. II and III
E. I, II and III
B. 500- 5,000mg/kg body wt
588. Hyperbaric oxygen supplementation may be necessary intervention if no response is seen with 100% oxygen supplementation in
C. 50- 500mg/kg body wt cases of poisoning with which of the following toxicants?

D. 5-50mg/kg body wt I. carbon monoxide

E. not more than 5 mg/kg body wt II. Nitrobenzene

582. The insecticide Azinphos-methyl can cause death when ingested at a dose of 20 mg/kg body wt. What is its toxicity rating? III. Cyanide

A.1 A. I only
B. II only
B. 2 C. I and III
D. II and III
C. 5 E. I, II and III
D. 5 589. Milk may be used to dilute ingested toxicants. Which of the following ingested poisons may be manage initially by the
administration of milk?
E. 6
I. Hypochlorite
583. Which of the following toxicants is known to accumulate slowly in the lungs by an active process and causes lung edema,
alveolitis, and progressive pulmonary fibrosis days to weeks after an exposure? II. Corrosive acids
A. Paraquat III. Fluoride

B. Scopolamine A. I only
B. II only
C. Parathion C. I and III
D. II and III
D. Dioxin E. I, II and III
E. Nicotine 590. Alteration of pH can alter renal elimination of certain toxins. Which of the following toxicants may be safely eliminated by either
acidification or alkalinization of urine?
I. Phencyclidine III. If respiratory depression develops, 2mg of naloxone is administered and may be repeated as necessary

II. Salicylates A. I only


B. II only
III. Amphetamines C. I and III
D. II and III
A. I only E. I, II and III
B. II only
C. I and III 597. A patient is given physostigmine to manage poisoning with anti-muscarinic agent. Which of the following is/are possible
D. II and III complications of therapy with physostigmine?
E. I, II and III
I. Cholinergic crisis
591. Which of the following rugs may be useful to manage complication of exposures to nitrogen oxides and nitrobenzene?
II. Seizures
A. Methylene blue
III. Asystole
B. Ethanol
A. I only
C. Calcium gluconate B. II only
C. I and III
D. Activated charcoal D. II and III
E. I, II and III
E. Penicillamine
598. In the management of Mercury Poisoning, which of the following is/are important considerations?
592. Toxidrome consisting of CNS depression followed by severe metabolic acidosis, blindness with initial “snowstorm-like” visual
disturbance, and coma is most consistent with which of the following toxicants? I. dimercaprol is not used for elemental and organic mercury poisoning as the drug may redistribute mercury from tissues to brain
A. Nitrates II. Chelation from unithiol may diminished nephrotoxicity that may result from acute exposure to inorganic mercury
B. Warfarin III. In acute exposure, the efficacy of chelation therapy diminishes with time since exposure
C. Ethanol A. I only
B. II only
D. Methanol C. I and III
D. II and III
E. Ethylene glycol
E. I, II and III
593. The manifestations of oxalate poisoning are best managed with which of the following antidotes?
599. Dimercaptopropanesulfonic acid is also known as ______?
A. Calcium gluconate
A. Dimercaprol
B. Fomepizole
B. Succimer
C. Glucagon
C. D-dimethylcysteine
D. Sodium bicarbonate
D. unithiol
E. Atropine
E. Deferoxamine
594. A 20 year old male patient is admitted after ingesting 20 tablets of 300mg Isoniazid tablet. Which of the following statements
600. What is the mechanism of action of Glucagon that makes the drug useful for the management of overdoses or poisoning with
is/are correct regarding the management of this patient?
drugs that directly inhibit myocardial contractility?
I. Administer a total of 6000mg pyridoxine in divided doses by slow IV
A. glucagon directly stimulates calcium release from the sarcoplasmic reticulum
II. Give activated charcoal slurry
B. Glucagon directly binds to beta adrenergic receptors leading to an increase cAMP
III. Given benzodiazepines to control seizures
C. Glucagon binds to glucagons receptors that stimulates production of IP3 and cause an increase in intracellular calcium
A. I only
D. Glucagon binds to glucagons receptors leading to generation of cAMP
B. II only
C. I and III E. Glucagon increases cAMP levels by inhibiting the enzyme phosphodiesterase
D. II and III
E. I, II and III

595. Historically, a “universal antidote” was employed in the management of poisoning. Recent studies have shown that the two of the
components of this so-called universal antidote have no significant efficacy. What are the components of this so-called “universal
antidote”?

A. activated charcoal, calcium oxide, tannic acid f.


B. activated charcoal, magnesium oxide, tannic acid

C. activated charcoal, milk of magnesia, milk

D. Milk of magnesia, tannic acid, strong tea

E. activated charcoal, milk, ipecac

596. Which of the following interventions is/are applicable in cases of acute opioid overdose?

I. Activated charcoal dose not have a role in the management of opoid poisoning

II. Naloxone can be administered IV, IM, or SC

You might also like